Đáp án thi chọn Đội Tuyển PTNK năm học 2012-2013

Kì thi chọn đội tuyển thi HSG Quốc gia của trường PTNK năm học 2012 – 2013 được diễn ra trong hai ngày, mỗi ngày 4 bài làm trong 180 phút.

Ngày thi thứ 1

Bài 1. Giải hệ phương trình:

$$ \begin{cases}x+y & =3 \\ x z+y t & =5 \\ x z^{2}+y t^{2} & =41 \\ x z^{3}+y t^{3} & =121\end{cases} $$

Bài 2.Cho dãy $\left(u_{n}\right)$ giảm và có giới hạn bằng 0 . Xét hai dãy số sau

$$ \begin{gathered} v_{n}=u_{1}+u_{2}+\cdots+u_{n}-n u_{n+1} \ z_{n}=u_{1}+u_{2}+\cdots+u_{n} \end{gathered} $$

Chứng minh nếu $\left(v_{n}\right)$ bị chặn thì $\left(z_{n}\right)$ hội tụ.

Bài 3. Cho tập $X={1 ; 2 ; 3 ; \ldots ; 4 n} .$ Hai tập con $A$ và $B$ của $X$ được gọi là không giống nhau nếu $|A \Delta B| \geq 2 n+1$. Trong đó $A \Delta B=(A \backslash B) \cup(B \backslash A)$. Cho $\left\{A_{1} ; A_{2} ; \ldots ; A_{m}\right\}$ là tạp con của $X$ gồm $m$ phần tử đôi một không giống nhau.

  1. Chứng minh $m \leq 2 n$.
  2. Chứng minh $m \leq \frac{4(n+1)}{3}$.

Bài 4. Cho $\triangle A B C$ với $M, N$ thuộc cạnh $B C$ sao cho $\angle B A M=\angle C A N=\alpha$ (M nằm giữa $B, N)$. Gọi I là trung điểm BC. Kẻ BH $\perp A M, C K \perp A N$ lần lượt tại $H, K$.

  1. Chứng minh tâm đường tròn (IHK) luôn thuộc một đường thẳng cố định.
  2. Tính $\alpha$ theo $\angle A B C$ và $\angle A C B$ sao cho $(I K H)$ tiếp xúc với đường tròn đường kính $\mathrm{AB}$ hoặc đường tròn đường kính $\mathrm{AC}$.

Ngày thi thứ 2

Bài 5.

  1. Chứng minh rằng với mọi $x>0$ thi $2\left(x^{\frac{4}{3}}+\frac{1}{x^{\frac{4}{3}}}+1\right) \geq 3\left(x+\frac{1}{x}\right)$.
  2. Tim số thực dương a nhỏ nhất sao cho $2\left(x^{a}+\frac{1}{x^{a}}+1\right) \geq 3\left(x+\frac{1}{x}\right)$ với mọi $x>0$

Bài 6. Tìm $n$ tự nhiên sao cho $A_{n}=1+3^{20\left(n^{2}+n+1\right)}+9^{14\left(n^{2}+n+1\right)}$ là số nguyên tố.

Bài 7. Tìm hàm số $f: N * \rightarrow N$ * thỏa mãn đồng thời hai điều kiện sau:

i) $f(m f(n))=n^{6} \cdot f(m n)$

ii) Với mọi $m, n$ là các số nguyên tố cùng nhau thì $f(m), f(n)$ cũng nguyên tố cùng nhau.

Bài 8. Cho tam giác $A B C$ có $A B=A C$. Gọi $H$ là chân đường vuông góc hạ từ A xuống $B C$. ( $C)$ là một đường tròn đi qua $H$ có tâm $I$ di động trên đoạn $A H .(C)$ cắt $A B$ tại $M, N$ và cắt $A C$ tai $P, Q$ sao cho $M$ nằm giữa $A$ và $N, P$ nằm giũa $A v \dot{a} Q$. $B P$ và $B Q$ cắt $(C)$ tại $E, F$. Chúng minh rằng giao điểm $D$ của $N E$ và $M F$ là một điểm cố định.

Giải

Ngày thi thứ 1

Bài 1.

Gọi 4 phương trình của hệ lần lượt là $(1), (2), (3), (4).$ Nếu $z+t=0$ thì khử $t$ ở các phương trình.

Ta có $x+y=3$ và $(x+y){{z}^{2}}=41$ nên ${{z}^{2}}=\frac{41}{3}$.

Mặt khác $z(x-y)=5$ và ${{z}^{3}}(x-y)=101$ nên ${{z}^{2}}=\frac{101}{5},$ từ đây ta có điều vô lý.

Do đó, ta chỉ cần xét $z+t \neq 0$. Từ $(2),$ ta có

$$(xz + yt)(z + t) = 5(z + t),$$

sau khi khai triển, sử dụng $(1),(3)$, ta được $$41 + 3zt = 5(z + t).$$

Từ $(3),$ ta có $$(x{z^2} + y{t^2})(z + t) = 41(z + t),$$

lại khai triển, sử dụng $(2),(4)$, ta được $$101 + 5zt = 41(z + t).$$

Do đó, ta có một hệ phương trình theo biến $zt,z+t$ nên giải ra được $z+t=1,zt=-12$. Từ đây ta có $(z,t)=(4,-3),(-3,4).$

  •  Nếu $z=4,t=-3$ thì dễ dàng tính được $x=2,y=1.$
  •  Nếu $z=-3,t=4$ thì ta cũng tính được $x=1,y=2.$

Vậy hệ phương trình có hai nghiệm phân biệt là $$(x,y,z,t)=(1,2,-3,4),(2,1,4,-3).$$

Bài 2.

Do $z_n$ là dãy tăng nên ta chỉ cần chứng minh nó bị chặn trên là được.

Giả sử $L$ là chặn trên của $v_n$, ta có do $x_n$ giảm nên

$${z_n} – n{x_{n + 2}} \le {v_{n + 1}} = {x_1} + {x_2} + \ldots + {x_n} + {x_{n + 1}} – (n + 1){x_{n + 2}} \le L.$$

Suy ra \[{z_n} \le L + n{x_{n + 2}}.\]

Tương tự, \[{z_n} – n{x_{n + 3}} \le {v_{n + 2}} = {x_1} + {x_2} + \ldots + {x_n} + {x_{n + 1}} + {x_{n + 2}} – (n + 2){x_{n + 3}} \le L\]

nên ta được \[{z_n} \le L + n{x_{n + 3}}.\]

Qua hai bước trên, làm tương tự, cho $n$ cố định, ta suy ra được\[{z_n} \le L + n\mathop {\lim }\limits_{N \to \infty } {x_N} = L.\]

Ta có điều phải chứng minh.

Bài 3.

(a) Ta giải bằng phương pháp đếm bằng hai cách. Giả sử ngược lại, tồn tại tập $M$ gồm $2n+1$ tập con đôi một không giống nhau. Ta đếm số các bộ $(x, i, j)$ với $i < j$ mà $x \in A_i \Delta A_j.$ Theo giả thiết, số bộ này ít nhất là

$(2n+1)\frac{(2n+1)2n}{2}.$

Với mỗi $x,$ giả sử có $k$ tập con $A_i$ chứa $x$ và $2n+1-k$ tập con không chứa $x.$ Khi đó số cặp $A_i, A_j$ mà $x \in A_i \Delta A_j$ sẽ không quá $k(2n+1-k) \le n(n+1).$ Như vậy số bộ này nhiều nhất là $4n \cdot n(n+1).$ Từ đó ta suy ra bất đẳng thức   \[(2n+1)\frac{(2n+1)2n}{2}\le 4n \cdot n(n+1).\]

Suy ra $1 ≤ 0,$ vô lý, kéo theo $m \le 2n.$

(b) Xét tập $M = \{A_1, A_2, \ldots, A_m \}$ gồm $m$ tập con đôi một không giống nhau của $X.$

Đặt $X’ = X \cup {0}.$ Ta thiết lập các tập $A’_i$ theo nguyên tắc sau đây

  •  Nếu $|A_i|$ chẵn thì $A’_i = A_i,$
  •  Nếu $|A_i|$ lẻ thì $A’_i = A_i \cup \{0\}.$

Khi đó ta có $|A’_i|$ chẵn với mọi $i.$ Từ đó, do

$$| A \Delta B | = | A | + | B | – 2| A \cap B | \text{ nên } |A’_i \Delta A’_j|.$$

Mà ${{{A}’}_{i}}\Delta {{{A}’}_{j}} \supseteq {{A}_{i}}\Delta {{A}_{j}}$ nên $\left| {{{{A}’}}_{i}}\Delta {{{{A}’}}_{j}} \right|\ge \left| {{A}_{i}}\Delta {{A}_{j}} \right|\ge 2n+1.$ Từ đây, do tính chẵn, suy ra $|A’_i \Delta A’_j| \ge 2n+2.$

Lại áp dụng lập luận tương tự (a) cho tập $M’ = \{A’_1, A’_2, \ldots, A’_m \}$ thuộc $X’,$ ta có số bộ $(x, i, j)$ với $x \in A’_i \Delta A’_j$ một mặt sẽ có ít nhất là $$(2n+2)\frac{m(m-1)}{2}.$$ Mặt khác sẽ không vượt quá $(4n+1) \cdot \frac{{{m}^{2}}}{4}.$ Từ đó suy ra bất đẳng thức

\[\begin{aligned} & (2n+2)\frac{m(m-1)}{2}\le (4n+1) \cdot \frac{{{m}^{2}}}{4} \\ &\Leftrightarrow 4(n+1)(m-1)\le (4n+1)m \\ & \Leftrightarrow m\le \frac{4(n+1)}{3}. \end{aligned}\]

Bài 4.

(a) Gọi $AT$ là đường cao của tam giác $ABC$ với $T \in BC.$ Suy ra tứ giác $AHTB$ và $ATKC$ nội tiếp.

Không có mô tả.

Do đó $$\angle{HTI}=\angle{BAH}=\alpha =\angle{CAK}=\angle{ITK}.$$ Suy ra $I,H,T,K$ đồng viên. Do đó, tâm $(IHK)$ thuộc trung trực của $TI$ cố định.

(b) Gọi $(\omega)$ là đường tròn đường kính $AB.$ Ta có tứ giác $AHTB$ nội tiếp $(\omega).$ Do đó $(\omega)$ giao $(IHK)$ tại $T$ và $H.$

Giả sử $(\omega)$ và $(IHK)$ tiếp xúc thì $T \equiv H$ dẫn đến $AM$ là đường cao của tam giác $ABC.$

Khi đó ta tính được $\alpha = 90^{\circ} -\angle{ABC}.$

Hoàn toàn tương tự nếu $(IHK)$ tiếp xúc với đường tròn đường kính $AC.$

Ngày thi thứ 2

Bài 5.

(a) Đặt $x^{\frac{4}{3}} = a, x^{-\frac{4}{3}}=b$ thì ta có $ab=1$ và cần chứng minh rằng

$$2(a^4+b^4+1) \ge 3(a^3+b^3).$$

Ta cũng có

$a^4+b^4=((a+b)^2-2)^2-2$ và $a^3+b^3=(a+b)^3-3(a+b)$.

Đặt $t = a+b \ge 2$ thì ta cần có

$2((t^2-2)^2-1) \ge 3(t^3-3t)$.

Biến đổi tương đương bất đẳng thức này, ta được

$$(t-2)(2t+1)(t^2-3) \ge 0.$$

Bất đẳng thức này đúng do $t \ge 2$.

(b) Ta chứng minh hằng số nhỏ nhất là $\alpha_0=\sqrt{3/2}.$ Không mất tính tổng quát, giả sử $x\ge 1.$ Đầu tiên, vì hàm $h(\alpha)=x^{\alpha}+\frac{1}{x^{\alpha}}$ tăng nên ta dễ thấy $\alpha>1.$ Bây giờ ta sẽ chứng minh bằng hai bước. \medskip

Bước : $\alpha=\sqrt{3/2}=\alpha_0$ thì bất đẳng thức đúng. Thật vậy, ta cần chứng minh

$$\frac{2(x^{\alpha_0}-1)^2}{x^{\alpha_0}}\ge\frac{3(x-1)^2}{x},$$

tương đương $f(x)=x^{\alpha_0}-\frac{1}{x^{\alpha_0}}-\alpha_0 x+\frac{\alpha_0}{x}.$

Ta có

$$f'(x)=\alpha_0 (x^{\alpha_0-1}-1)\left(1-\frac{1}{x^{\alpha_0+1}}\right)\ge 0 \forall x\ge 1,$$

nên $f(x)\ge f(1)=0.$

Bước 2: Chứng minh $\alpha=\sqrt{3/2}=\alpha_0$ là hằng số nhỏ nhất. Thật vậy, giả sử tồn tại $1<\alpha<\alpha_0$ sao cho bất đẳng thức ở đề bài thỏa mãn với mọi $x\ge 1.$ Khi đó tồn tại $m>n$ nguyên dương sao cho $\alpha<\frac{m}{n}<\alpha_0.$ Khi đó $h(\alpha)<h\left(\frac{m}{n}\right).$ Từ đây suy ra

$$2h\left(\frac{m}{n}\right)+2\ge3h(1),$$

tương đương

$$\frac{2(y^m-1)^2}{y^m}\ge\frac{3(y^n-1)^2}{y^n} \, \forall y\ge 1,$$

với $y=\sqrt[n]{x}.$ Bất đẳng thức trên có thể được viết lại với $y>1$ như sau

$$g(y)=2(y^{m-1}+y^{m-2}+\cdots+1)^2-3y^{m-n}(y^{n-1}+y^{n-2}+\cdots+1)^2\ge 0, \, \forall y>1.$$

Vì $g(y)$ liên tục nên ta có

$$\lim_{y\to 1}g(y)\ge0 \iff 2m^2\ge3n^2,$$

mâu thuẫn do $m/n<\sqrt{3/2}.$

Vậy hằng số nhỏ nhất cần tìm là $\sqrt{3/2}.$

Bài 6.

Đặt $a = 3^{4(n^2+n+1)} \ge 81$ thì ta có $A=t^7+t^5+1 $. Phân tích biểu thức này thành nhân tử, ta có

$$A=(t^5-t^4+t^3-t+1)(t^2+t+1).$$

Dễ dàng chứng minh được $$t^5-t^4+t^3-t+1 > t^2+t+1 > 1$$ nên biểu thức trên không thể là số nguyên tố được.

Vậy không tồn tại số $n$ nào thỏa mãn đề bài.

Bài 7.

Trong $(i),$ thay $m=1$, ta có

$$f(f(n))=n^6f(n).(*)$$

Từ đây dễ suy ra $f$ đơn ánh. Tiếp tục thay $m$ bởi $f(m),$ ta có

$$f(f(m) \cdot f(n))=n^6 \cdot f(f(m) \cdot n)=(mn)^6 \cdot f(mn)=f(f(mn)).$$

Kết hợp với tính đơn ánh, ta có $ f(m) \cdot f(n)=f(mn)$ nên $f$ nhân tính. \medskip

Tiếp theo, giả sử tồn tại $n$ sao cho $\gcd (f(n),n)=1$ suy ra $f(f(n))$ và $f(n)$ nguyên tố cùng nhau. Từ $(*)$ ta được $f(n)=1$ và $f(1)=n^6$. Lại có $\gcd (f(1),1)=1$ nên $f(1)=1$ . Do đó $$\gcd (f(n),n) \neq 1,\forall n \ge 2.$$

Xét số nguyên $n$ bất kì. Giả sử $f(n)$ có ước nguyên tố $q$ sao cho $q$ không là ước của $n.$ Khi đó $\gcd (n,q)=1$ và $\gcd (f(n),f(q)) \neq 1, $ trái với điều kiện $(ii).$ \medskip

Xét số nguyên tố $p$ bất kì. Khi đó $f(p)=p^k, k \ge 1$ do $(f(p),p) \neq 1$. Suy ra $$f(f(p))=f(p^k)=f(p)^k= p^{k^2}.$$

Thay vào $(*)$ ta được $$p^{k^2}=p^6 \cdot p^k \Rightarrow \begin{cases} k=3 \\ f(p)=p^3 \end{cases}.$$

Từ đây, kết hợp với tính nhân tính của $f$ và ta cũng biết rằng mỗi số $n$ đều có thể biểu diễn thành tích của các số nguyên tố, ta có được $f(n)=n^3$ với mọi $n \in \mathbb{N}.$

Bài 8.

Giả sử $MF$ cắt $BC$ tại $T$, ta có $(C)$ tiếp xúc với $BC$ tại $H$ nên $$TH^2=TF \cdot TM.$$

Không có mô tả.

 

Ta lại có $NQ \parallel BC$ vậy nên $$\angle{TBF}=\angle{FQN}=\angle{TMB},$$ suy ra $TB^2=TF \cdot TM.$ Từ đó, suy ra $T$ là trung điểm $HB$. Tương tự, ta cũng chứng minh được rằng $NE$ cắt $BC$ cũng tại trung điểm $BH$.

Suy ra $NE, MF, BC$ đồng quy tại $T$ là trung điểm $BH$, là một điểm cố định.

Nhận xét: Bài toán này cũng có một phiên bản tương tự khi thay tam giác tam giác cân bởi tam giác bất kỳ, và đường tròn cắt tại hai điểm trên cạnh thành đường tròn nội tiếp. Cụ thể là:

Cho tam giác $ABC$ có đường tròn nội tiếp $(I)$ tiếp xúc với các cạnh $BC,CA,AB$ lần lượt tại $D,E,F.$ Đường thẳng qua $E,F$ song song với $BC$ lần lượt cắt lại $(I)$ tại $G,H$. Giả sử $BG,CH$ cắt lại $(I)$ tại $P,Q$ và $EF$ cắt $BC$ tại $T$.

a/ Chứng minh rằng $PE,QF$ cùng đi qua trung điểm $K$ của $DT.$

b/ Chứng minh rằng $(KBP),(KCQ)$ cùng tiếp xúc với $(I)$ và hai đường tròn này cũng cắt nhau tại một điểm nằm trên $(ABC)$.

Đề thi chọn đội tuyển PTNK năm học 2010-2011

Ngày thi thứ 1

Bài 1.  Giải hệ phương trinh: $\left\{\begin{array}{l}\frac{5(x+y)}{x+y+6 x y}+\frac{6(x+z)}{x+z+5 x z}=4 \\ \frac{6(z+y)}{z+y+4 z y}+\frac{4(x+y)}{x+y+6 x y}=5 \\ \frac{4(x+z)}{x+z+5 x y}+\frac{5(y+z)}{y+z+4 y z}=6\end{array}\right.$

Bài 2.  Tìm tất cả các hàm $f: \mathbb{R} \rightarrow \mathbb{R}$ thỏa mãn

$f(|x|+y+f(y+f(y)))=3 y+|f(x)|$ với mọi $x, y \in \mathbb{R}$

Bài 3.  Cho $p$ là số nguyên tố lẻ và $n=2 p+r$ với $r \in{0,1,2, \ldots, p-1} .$ Đặt $X={1,2, \ldots, n}$. Ánh xạ $f: X \rightarrow X$ được gọi là có tính chất $P$ nếu $f$ không phải là ánh xạ đồng nhất và $f(f \ldots(f(f(k))) \ldots)=k$ (ánh xạ hợp plần) với mọi $k \in X .$ Đạt $A_{f}={k \in X \mid f(k)=k}$.

  1. Chứng minh rằng nếu $f$ có tính chất P thì $\left|A_{f}\right| \equiv r(\bmod p)$.
  2. Gọi $d$ là số các ánh xạ $ f$ có tính chất $P$. Chứng minh rằng $d$ không là ước số của n!.

Bài 4.  Cho tam giác $A B C$ nội tiếp đường tròn $(O)$ có $A$ cố định và $B, C$ thay đổi trên $(O)$ sao cho $B C$ luôn song song với một đường thẳng cố định. Các tiếp tuyến của $(O)$ tại $B$ và $C$ cắt nhau tại $K$. Gọi $M$ là trung điểm $B C, N$ là giao điểm của $A M$ với $(O)$. Chứng minh rằng đường thẳng $K N$ luôn đi qua một điểm cố định.

Ngày thi thứ 2

Bài 5.  Cho $a, b, c$ là độ dài các canh của một tam giác. Chứng minh rằng $(2 a+2 b-c)(2 b+2 c-a)(2 a+2 c-b)>25 a b c$

Bài 6.  Cho dãy số $u_{n}$ thỏa mãn $u_{1}=\sqrt{2}, u_{n+1}=\frac{2 u_{n}^{2}+5 u_{n}+5}{2 u_{n}+4}, n \geq 1$. Tính giới hạn sau $\lim \frac{u_{n}^{2}-3 u_{n}+5}{3 n^{2}+4 n-1}$.

Bài 7.  Xét số tự nhiên $n>1$. Bắt đầu từ bộ số $1,2, \ldots, 2 n-1,2 n$ ta thực hiện phép biến đổi sau: chọn hai số $a, b$ bát kì sao $a-b>1$, xóa hai số này và thay bởi hai số $a-1$ và $b+1$. Với bộ số mới này ta lại tiếp tục thực hiện phép biến đổi tương tự.

  1. Chứng minh rằng ta sẽ đạt đến trạng thái dừng, tức là không thể thực hiện phép biến đổi như vậy được nữa.
  2. Gọi $k$ là số lần phép biến đổicần thực hiện để đạt đến trang thái dừng. Tìm giá trị nhỏ nhất và lớn nhất của $k$.

Bài 8.  Cho đường tròn $\left(\gamma_{1}\right)$ dường kính $A B$ và đường tròn $\left(\gamma_{2}\right)$ tâm $A$ cắt $\left(\gamma_{1}\right)$ tai $C$ và $D$. Điểm $M$ thay đổi trên cung $C D$ (nằm bên trong $\left.\left(\gamma_{1}\right)\right)$ của $\left(\gamma_{2}\right)$, $B M$ cắt $\left(\gamma_{2}\right)$ tại $N$ (N khác $M$ và B). Tìm giá trị nhỏ nhất của $\frac{N C+N D}{N M}$.

Lời giải

Bài 1. Đặt $u=\frac{x+y}{x+y+6xy},v=\frac{y+z}{y+z+4yz},w=\frac{z+x}{z+x+5zx}$ thì ta có hệ
$$\left\\{ \begin{aligned}
& 5u+6w=4 \\\\
& 6v+4u=5 \\\\
& 4w+5v=6 \\\\
\end{aligned} \right.\Leftrightarrow \left\\{ \begin{aligned}
& 8u=1 \\\\
& 4v=3 \\\\
& 16w=9 \\\\
\end{aligned} \right..$$
Suy ra
$$\left\\{ \begin{aligned}
& 7(x+y)=6xy \\\\
& 3(y+z)=12yz \\\\
& 7(z+x)=45zx \\\\
\end{aligned} \right.\Leftrightarrow \left\\{ \begin{aligned}
& a+b=\frac{6}{7} \\\\
& b+c=12 \\\\
& c+a=\frac{45}{7} \\\\
\end{aligned} \right.,$$ trong đó $a=\frac{1}{x},b=\frac{1}{y},c=\frac{1}{z}.$
Giải hệ trên, ta thu được $a=-\frac{33}{14},b=\frac{45}{14},c=\frac{123}{14}$ nên $(x,y,z)=\left( -\frac{14}{33},\frac{14}{45},\frac{14}{123} \right).$

Bài 2.

Dễ thấy $f$ toàn ánh. Giả sử $f(a)=0$ và thay $x=0,y=a,$ ta có
$$0=3a+\left| f(0) \right|.$$
Suy ra $a$ tồn tại duy nhất và $a=-\frac{1}{3}\left| f(0) \right|\le 0.$ Lại thay $x=y=a,$ ta có $f(0)=3a\le 0.$
Lại thay $x=-a,y=a$ thì chú ý rằng $\left| -a \right|+a=0$, ta có $f(0)=3a+\left| f(-a) \right|$ nên $f(-a)=0$, điều này kéo theo $a=-a$ hay $a=0$ (do tính duy nhất ở trên). \medskip

Thay $y=0$ thì $f(\left| x \right|)=\left| f(x) \right|$ nên $f(x)\ge 0,\forall x\ge 0.$
Xét $x>0$ và $y=-\frac{f(x)}{3}$, ta có $f\left( x-\frac{f(x)}{3}+f\left( -\frac{f(x)}{3}+f\left( -\frac{f(x)}{3} \right) \right) \right)=0$ nên
$$-\frac{f(x)}{3}+f\left( -\frac{f(x)}{3}+f\left( -\frac{f(x)}{3} \right) \right)=-x$$ với mọi $x>0.$
Trong đề bài, thay $x=0$ thì $f(y+f(y+f(y)))=3y$. Thay $y\to -\frac{f(x)}{3}$ thì
$f\left( -\frac{f(x)}{3}+f\left( -\frac{f(x)}{3}+f\left( -\frac{f(x)}{3} \right) \right) \right)=-f(x)$.
So sánh hai đẳng thức trên, ta có $f(-x)=-f(x),\forall x>0$ nên $f$ là hàm số lẻ. \medskip

Từ tính chất hàm số lẻ, ta có $f\left( \frac{f(x)}{3}+f\left( \frac{f(x)}{3}+f\left( \frac{f(x)}{3} \right) \right) \right)=f(x)$ với mọi $x>0.$
Trong đề bài, xét $x\ge 0$ và $y\to \frac{f(y)}{3}$, ta có
$$f\left( x+\frac{f(y)}{3}+f\left( \frac{f(y)}{3}+f\left( \frac{f(y)}{3} \right) \right) \right)=f(y)+f(x)$$ hay
$f(x+y)=f(x)+f(y)$ với mọi $x,y>0.$
Vì $f$ cộng tính trên ${{\mathbb{R}}^{+}}$ nên ta có $f(x)=ax,\forall x>0.$ Lại do tính chất hàm lẻ, ta suy ra $f(x)=ax,\forall x\in \mathbb{R}.$ Thay vào đề bài, ta có $a=1.$ \medskip

Vậy tất cả các hàm số cần tìm là $f(x)=x.$

Bài 3.

a) Ta có
$$\left | A_{f} \right |\equiv r \pmod{p} \Leftrightarrow \left | X\setminus A_{f} \right | \text{ chia hết cho } p.$$
Điều này tương đương số phần tử của tập hợp $B= \left \\{ k\in X|f(k)\neq k \right \\}$ là bội của $p.$
Đặt $f_{m}(x)$ là ánh xạ hợp $m$ lần. Xét $x\in B$ thì cũng có các số $f(x),f_{2}(x),\ldots, f_{p-1}(x)\in B$. Thật vậy, \medskip

Gỉa sử tồn tại $1<m<p$ sao cho $f_{m}(x) =x$ với số $x \in B$ nào đó, ta chọn $m$ là số nhỏ nhất như thế. Vì $p$ nguyên tố lẻ nên $p$ không chia hết cho $m.$ Do vậy tồn tại số $t$ sao cho $0<p-tm<m$. Lại có $$f_{m}(x) =x\Rightarrow f_{tm}(x)=x\Rightarrow f_{p-tm}(x)=f_{p}(x)=x$$ (mâu thuẫn với tính nhỏ nhất của $m$). Vì thế nên với mọi $m$ mà $1<m<p $ thì $f_{m}(x)\neq x$. Từ đó suy ra với mọi $1<k<l<p$ thì $f_{k}(x)\neq f_{l}(x) $, tức là $x,f(x),f_{2}(x),\ldots, f_{p-1}(x) $ là $p$ số khác nhau thuộc $B.$ \medskip

Xét số $y \in B$ và $y$ khác tất cả $p$ số ở trên. Khi đó, ta cũng sẽ có $y$ sinh ra một bộ $p$ số phân biệt mới. Giả sử rằng có $f_{i}(x)=f_{j}(y)$ với $i<j$ nào đó thì sẽ có $f_{p+i-j}(x)=f_{p}(y)=y $, mâu thuẫn. Suy ra trong $B$ sẽ có $1$ hoặc $2$ bộ $p$ số rời nhau, chứng tỏ rằng số phần tử của $B$ chia hết cho $p.$ Suy ra đpcm. \medskip

(b) Từ đây ta thấy rằng để đếm số ánh xạ $f$ có tính chất $\mathcal{P},$ trước hết, ta chọn ra $r$ hoặc $p+r$ vị trí cố định. Ta xét hai trường hợp như sau:

Nếu $\left| A_f \right|=p+r$ thì có $C_n^{p+r}$ cách chọn ra các số này, còn lại $p$ số thì $f$ phải là song ánh trên tập con đó. Do đó trong trường hợp này có $p!C_n^{p+r}$ cách.
Nếu $\left| A_f \right|=r$ thì tương tự trên, ta cũng đếm được $(p!)^2C_n^rC_{2p}^p$.

Từ đó suy ra số ánh xạ tính chất $\mathcal{P}$ là $$d=p!C_{n}^{p+r}+{{(p!)}^{2}}C_{n}^{r}C_{2p}^{p}.$$ Ta sẽ chứng minh số này không là ước của $n!.$ Ta viết số $d$ dưới dạng khai triển
$$d=p!\frac{n!}{(p+r)!p!}+{{(p!)}^{2}}\frac{n!}{r!(2p)!}\cdot \frac{(2p)!}{{{(p!)}^{2}}}=\frac{n!}{(p+r)!}+\frac{n!}{r!}.$$
Đặt $(p+r)!=k\cdot {{(r!)}^{2}}$ với $k=\frac{(p+r)!}{{{(r!)}^{2}}}=\frac{p!}{r!}\cdot \frac{(p+r)!}{p!r!}=\frac{p!}{r!}C_{p+r}^{r}\in \mathbb{Z}$. Khi đó, ta viết lại
$$\frac{n!}{d}=\frac{r!(p+r)!}{r!+(p+r)!}=\frac{k\cdot {{(r!)}^{3}}}{(1+k\cdot r!)\cdot r!}=\frac{k\cdot {{(r!)}^{2}}}{k\cdot r!+1}.$$
Dễ thấy số này không thể nguyên vì $k\cdot r!+1$ nguyên tố cùng nhau với $k\cdot {{(r!)}^{2}}$. Từ đó ta có $d$ không là ước của $n!.$

Bài 4. Giả sử $KN$ cắt $(O)$ tại $I$ thì tứ giác $BNCI$ điều hòa.

Do đó $A(BC,NI)=-1,$ mà $AN$ chia đôi $BC$ nên $AI\parallel BC,$ tức là $AI$ có phương cố định. Từ đó ta thấy $I$ là điểm cố định cần tìm.

Bài 5. Đặt $a+b-c=x,b+c-a=y,c+a-b=z$ thì $x,y,z>0.$ Ta đưa về bất đẳng thức
$$\left( 4\cdot \frac{x}{y+z}+1 \right)\left( 4\cdot \frac{y}{z+x}+1 \right)\left( 4\cdot \frac{z}{x+y}+1 \right)>25.$$
Không mất tính tổng quát, giả sử $0<x\le y\le z.$ Đặt $S=x+y+z.$ Ta đưa về
$$(S+3x)(S+3y)(S+3z)>25(S-x)(S-y)(S-z).$$
Khai triển và rút gọn, ta được
$${{S}^{3}}-4S(xy+yz+zx)+13xyz>0.$$
Chú ý rằng $${{S}^{3}}-4S(xy+yz+zx)=S({{S}^{2}}-4(xy+yz+zx))=S({{(x+y-z)}^{2}}-4xy)$$ nên ta đưa về $S{{(x+y-z)}^{2}}+xy(13z-4S)>0$.
Bất đẳng thức cuối đúng vì $13c-4S=9z-4(x+y)>0.$

Bài 6.

Lời giải. Ta thấy rằng $u_{n}>0, \forall n$ và $u_{n+1}-u_{n}=\frac{u_{n}+5}{2\left(u_{n}+2\right)}>0$ nên dãy tăng. Giả sử dãy bị chặn trên thì nó hội tụ về $L>0$, suy ra
$$
L=\frac{2 L^{2}+5 L+5}{2 L+4} \Leftrightarrow L=-5
$$
vô lý. Suy ra $\lim_{n \rightarrow+\infty} u_{n}=+\infty$. Từ đó, ta được
$$
\lim_{n \rightarrow+\infty}\left(u_{n+1}-u_n\right)=\lim_{n \rightarrow+\infty} \frac{u_n+5}{2\left(u_n+2\right)}=\frac{1}{2}
$$
nên theo định lý Stolz, ta suy ra $\lim_{n \rightarrow+\infty} \frac{u_n}{n}=\frac{1}{2}$ và $\lim_{n \rightarrow+\infty} \frac{u_n}{n^2}=0 .$ Do đó, trong biểu thức cần tính giới hạn, chia tử và mẫu cho $n^2$ rồi áp dụng kết quả trên, ta có
$$
\lim_{n \rightarrow+\infty} \frac{u_n^2-3 u_n+5}{3 n^2+4n-1}=\lim _{n \rightarrow+\infty} \frac{\left(\frac{u_n}{n}\right)^{2}-\frac{3 u_n-5}{n^2}}{3+\frac{4}{n}-\frac{1}{n^2}}=\left(\frac{1}{2}\right)^{2} \cdot \frac{1}{3}=\frac{1}{12}
$$

Bài 7.

(a) Xét đại lượng $S$ là tổng bình phương các số thu được sau mỗi thao tác biến đổi. \medskip

Ta thấy rằng từ $(a,b)$ với $a-b>1,$ ta đưa về bộ $(a-1,b+1)$ thì tổng trên thay đổi một lượng là
${{a}^{2}}+{{b}^{2}}-{{(a-1)}^{2}}-{{(b+1)}^{2}}=2(a+b-1)>0.$
Do đó, tổng $S$ giảm ngặt, và rõ ràng $S$ phải luôn dương nên thao tác trên chỉ thực hiện được trong hữu hạn lần.
\medskip

(b) Rõ ràng tổng trên không đổi khi không còn cặp số $a,b$ nào mà $a-b>1.$ Điều này đồng nghĩa với việc các số thu được trong trạng thái cuối chỉ nhận hai giá trị liên tiếp nào đó. Ta thấy rằng tổng các số đã cho luôn không đổi và là $1+2+\cdots +2n=n(2n+1).$ \medskip

Giả sử cuối cùng, ta có $x$ số $m$ và $y$ số $m+1$ thì
$$\left\\{ \begin{aligned}
& x+y=2n \\\\
& mx+(m+1)y=n(2n+1) \\\\
\end{aligned} \right..$$
Suy ra $2mn+y=2{{n}^{2}}+n\Rightarrow n|y.$ Tuy nhiên, nếu $y\in \{0,2n\}$ thì vô lý vì vế phải không chia hết cho $2n.$ Do đó $x=y=n$ và $m=n,$ tức là ở trạng thái cuối, ta còn $n$ số $n$ và $n+1.$

  • Tổng bình phương của chúng là $S=n\cdot {{n}^{2}}+n\cdot {{(n+1)}^{2}}=n(2{{n}^{2}}+2n+1).$
  • Tổng bình phương ban đầu là ${{S}_{0}}={{1}^{2}}+{{2}^{2}}+\cdots +{{(2n)}^{2}}=\frac{n(2n+1)(4n+1)}{3}.$

Suy ra ${{S}_{0}}-S=\frac{2}{3}({{n}^{3}}-n).$ \medskip

(b) Để thực hiện được nhiều lần nhất thì giá trị giảm đi ở mỗi lần phải ít nhất. Theo câu a) thì giá trị đó sẽ là $2(a+b-1)\ge 2.$ \medskip

Suy ra số lần nhiều nhất sẽ là $\frac{1}{3}({{n}^{3}}-n)$. Để thực hiện được điều này, ta sẽ cố gắng trong mỗi thao tác tạo ra nhiều giá trị nhất có thể và đồng thời làm giảm số lượng các giá trị ở hai biên đi. Từ đó ta được ${{k}_{\max }}=\frac{1}{3}({{n}^{3}}-n).$ \medskip

Để thực hiện được ít lần nhất, ta sử dụng ý tưởng tham lam, mỗi lần, ta sẽ chọn các cặp số nằm về hai phía của $n,n+1.$ Khi đó, giá trị của các số $1,2,\ldots ,n-1$ sẽ dần dần được tăng lên, trong khi giá trị của các số $n+2,n+3,\ldots ,2n$ dần dần sẽ giảm đi. Tổng khoảng cách từ các số nhỏ hơn $n$ đến $n$ là $1+2+\cdots +n-1=\frac{n(n-1)}{2}$. Tương tự thì tổng khoảng cách các số lớn hơn $n+1$ đến $n+1$ cũng là $\frac{n(n-1)}{2}$. Ta thấy mỗi lần thao tác thì các số này sẽ thu hẹp khoảng cách đúng $2$ đơn vị nên số lần thao tác tối thiểu phải là $\frac{1}{2}\left( \frac{n(n-1)}{2}+\frac{n(n-1)}{2} \right)=\frac{n(n-1)}{2}.$ \medskip

Để đạt được giá trị này, mỗi lần, ta chỉ cần chọn các cặp số có dạng $(t,2n+1-t)$ với $1\le t\le n-1$ là được. Suy ra ${{k}_{\min }}=\frac{n(n-1)}{2}.$

Bài 8.

Theo định lý Ptolemy cho tứ giác $BCND$ nội tiếp trong ${{\gamma }_{1}}$ thì
$$BC\cdot ND+BD\cdot NC=BN\cdot CD.$$
Vì $AC=AD$ nên $BC=BD=m$ và $CD=n$ là các giá trị cố định.

Ta có
$$m(NC+ND)=n\cdot BN\Rightarrow NC+ND=\frac{n}{m}\cdot BN.$$
Suy ra $\frac{NC+ND}{MN}=\frac{n}{m}\cdot \frac{BN}{MN}.$ Ta đưa về tìm giá trị nhỏ nhất của $\frac{BN}{MN}.$ \medskip

Xét phương tích từ $B$ đến ${{\gamma }_{2}}$ thì $BM\cdot BN=BK\cdot BA=c$ là hằng số nên
$(BN-MN)BN=c$.
Do đó $\frac{MN}{BN}=1-\frac{c}{B{{N}^{2}}}$ nên
$$\frac{BN}{MN}\min \Leftrightarrow \frac{MN}{BN}\max \Leftrightarrow \frac{c}{B{{N}^{2}}}\min \Leftrightarrow B{{N}^{2}}\max .$$
Dễ thấy $\max BN=AB$, xảy ra khi $N\equiv A$ hay $M\equiv K.$ Khi đó $\frac{NC+ND}{MN}=\frac{AC+AD}{AK}=2$ chính là giá trị nhỏ nhất cần tìm.

Giới thiệu sách: Đề thi học sinh giỏi các nước.

Gửi các bạn một số Booklet của các nước.

1/ Đề thi Trung Quốc. các năm 2010, 2011 – 2014 có sách bản quyền. Các năm về sau chưa thấy sách.

Mathematical_Olympiad_in_China-Problems_and_Solutions 2003 2006

mathematic-olympiad-in-china2007-2008-problems-and-solutions

2/ Đề thi Bulgari. 

Một số sách đề thi Bulgari có nhiều bài hay được lấy làm đề thi tuyển sinh vào 10, đề thi học sinh giỏi trong nước.

Bulgarian Mathematical Competitions 2003-2006

BulgarianMO1960_2008

3/Đề thi khu vực Balkan https://vi.wikipedia.org/wiki/Balkan

Balkan MO 2008-20 EN with solutions

(Còn nữa)

Phương pháp chứng minh phản chứng (P2)

Bài 1. 

Cho tập $B = {1, 2, 3, …, 16}$. Người ta ghi các số của tập B thành một vòng tròn (mỗi số ghi một lần). Hỏi có cách ghi để tổng thỏa:

a/ Tổng của hai số kế nhau bất kì lớn hơn hoặc bằng 17 được không? Tại sao?

b/ Tổng của ba số kế nhau bất kì lớn hơn 24 được không? Tại sao?

Lời giải

a/ Giả sử tồn tại cách ghi thỏa mãn. Khi đó, gọi 2 số kề với 1 là a và b.

Theo giả thiết, ta có:

$\left\{\begin{array}{l} 1 + a \geqslant 17  \\1 + b \geqslant 17  \end{array} \right. \Rightarrow \left\{\begin{array}{l}  a \geqslant 16 \\ b \geqslant 16 \end{array} \right. \Rightarrow$ Mâu thuẫn.

Vậy không tồn tại cách ghi thỏa mãn.

b/ Giả sử tồn tại cách ghi thỏa mãn.

Khi đó, ta tách số 16 ra và chia 15 số còn lại thành 5 bộ 3 số kề nhau. Và tổng của 16 số này phải lớn hơn hoặc bằng: $16+5\cdot 25=141$

Mà $1+2+3+\cdots 16=136 \Rightarrow $ Mâu thuẫn

Vậy không tồn tại cách ghi thỏa mãn.

Bài 2. 

Có tồn tại hay không một cách điền các số $0,1, 2, 3, \cdots , 9$ vào các đỉnh của một đa giác 10 đỉnh sao cho hiệu hai số ở hai đỉnh kề nhau chỉ có thể nhận một trong các giá trị sau: $-5, -4, -3, 3, 4, 5$.

Lời giải

Giả sử có cách xếp thỏa mãn.

Xét các số $0,1,2,8,9$ không thể đứng kề nhau và có đúng 10 số nên 5 số còn lại phải đứng xen kẽ với 5 số $0,1,2,8,9$.

Xét số 7:

Khi đó hai số kề số 7 phải thuộc tập hợp $\left\{0,1,2,8,9\right\}$

Mà theo giả thiết 2 đỉnh kề nhau bất kì nhận một trong các giá trị – 3, – 4, – 5, 3, 4 hoặc 5 nên 2 số kề nhau với 7 đều bằng 2 $\Rightarrow$ Mâu thuẫn.

Vậy không có cách xếp nào thỏa mãn.

Bài 3. 

Trong mặt phẳng tọa độ thì một điểm mà hoành độ và tung độ đều là các số nguyên được gọi là điểm nguyên. Chứng minh rằng không tồn tại tam giác đều nào mà các đỉnh đều là điểm nguyên.

Lời giải

Giả sử tồn tại tam giác đều có các đỉnh là các điểm nguyên.

Xét hình chữ nhật có các đỉnh là các điểm nguyên, sao cho đỉnh của tam giác đều thuộc cạnh của hình chữ nhật. Khi đó dễ dàng suy ra diện tích tam giác đều là số hữu tỷ.

Ta có diện tích tam giác đều $S=\dfrac{a^{2} \sqrt{3}}{4}$ với $a^2=x^2+y^2$ là số nguyên, $\sqrt{3}$ là số vô tỷ

Do đó, S là số vô tỉ $\Rightarrow$ Mâu thuẫn $\Rightarrow$ đpcm.

Bài 4. 

Điền các số 1,2,3,…,121 vào một bảng ô vuông kích thước $11 \times 11$ sao cho mỗi ô chứa một số. Tồn tại hay không một cách điền sao cho hai số tự nhiên liên tiếp sẽ được điền vào hai ô có chung một cạnh và các tất cả các số chính phương thì nằm trong cùng một cột?

Lời giải

Giả sử tồn tại một cách điền số vào các ô thỏa yêu cầu đặt ra.

Khi đó bảng ô vuông được chia thành hai phần ngăn cách nhau bởi cột điền các số chính phương. Một phần chứa $11n$ ô vuông $1 \times 1$, và phần còn lại chứa $110-11n$ ô vuông $1 \times 1$ , với $0 \le n \le 5.$

Nhận thấy rằng các số tự nhiên nằm giữa hai số chính phương liên tiếp $a^2$ và $(a+1)^2$ sẽ cùng nằm về một phần và các số tự nhiên nằm giữa $(a+1)^2$ và $(a+2)^2$ sẽ nằm ở phần còn lại.

Ta có số lượng các số tự nhiên nằm giữa 1 và 4, 4 và 9, 9 và 16,…,100 và 121 lần lượt là $2,4,6,8,…,20$.

Do đó một phần sẽ chứa $2+6+10+14+18=50$ số, phần còn lại chứa $4+8+12+16+20=60$ số.

Cả 50 và 60 đều không chia hết cho 11 $\Rightarrow$ Mâu thuẫn.

Vậy không tồn tại cách điền số thỏa yêu cầu đề bài.

Bài 5. 

Mỗi phần tử của bảng vuông $ 25 \times 25 $ hoặc là $ + 1 $ hoặc $ -1 $. Gọi $ a_{i} $ là tích của tất cả các phần tử của hàng thứ $ i $ và $ b_{j} $ là tích của tất cả các phần tử của cột thứ $ j $. Chứng minh rằng $ a_ {1} + b_ {1} + \cdots + a_ {25} + b_ {25} \neq 0 $

Lời giải

Giả sử $ a_ {1} + b_ {1} + \cdots + a_ {25} + b_ {25} = 0 $.

Vì mỗi ô vuông chứa -1 hoặc 1 nên $a_i,b_i\in \left\{1,-1\right\}$

Do đó trong 50 tích $a_i,b_i\quad (i=\overline{1,25})$ sẽ có 25 tích có giá trị -1 và 25 tích có giá 1.

Khi thay thế một phần tử -1 trong bảng bằng 1 thì số các tích ngang dọc có giá trị -1 sẽ tăng 2 hoặc giảm 2 hoặc không thay đổi. Như vậy số các tích $a_i,b_i$ có giá trị -1 luôn là số lẻ (1)

Ta sẽ tiếp tục thay thế các phần tử -1 trong bảng bằng 1 cho đến khi tất cả các phần tử trong bảng đều bằng 1 thì khi đó số các tích ngang dọc $a_i,b_i$ có giá trị -1 là 0 $\Rightarrow$ Mâu thuẫn với (1) $\Rightarrow$ đpcm.

Bài tập số chính phương – Lớp 9

Bài 1. Chứng minh rằng

a) Một số chính phương chia 3 dư 0 hoặc 1.
b) Một số chính phương chia 4 dư 0 hoặc 1.
c) Một số chính phương chia 5 dư 0, 1 hoặc 4.
Bài 2. Chứng minh rằng một số là số chính phương khi và chỉ khi số ước của số đó là một số lẻ.

Bài 3. Chứng minh rằng nếu tổng hai số chính phương chia hết cho 3 thì tích của nó sẽ chia hết 81.

Bài 4. Chứng minh rằng với $n$ là số tự nhiên thì $3n-1, 5n + 2, 5n – 2, 7n-2, 7n+3$ không phải là số chính phương.

Bài 5. Tìm tất cả các số tự nhiên $n$ sao cho $n.2^{n+1}+1$ là một số chính phương.

Bài 6. Chứng minh rằng nếu $x^2+ 2y$ là một số chính phương với $x, y$ nguyên dương thì $x^2+ y$ là tổng của hai số chính phương.

Bài 7. Chứng minh rằng nếu $3x + 4y,3y + 4x$ là các số chính phương thì $x,y$ đều chia hết cho 7.

Bài 8. Cho các số nguyên dương $a, b$. Giả sử các số $a + 2b,b + 2a$ đều là bình phương của một số nguyên thì $a$ và $b$ đều chia hết cho 3.

Bài 9. Cho các số tự nhiên $a, b, c$ thỏa: $a + 2b,b + 2c,c + 2a$ đều là bình phương của một số tự nhiên.
a)Chỉ ra một bộ số thỏa đề bài.
b) Giả sử trong 3 số $a + 2b,b + 2c,c + 2a$ có một số chia hết cho 3. Chứng minh rằng: $P = \left( {a – b} \right)\left( {b – c} \right)\left( {c – a} \right)$ chia hết cho 27.

Bài 10. Chứng minh rằng nếu $\overline {abc} $ là một số nguyên tố thì ${b^2} – 4ac$ không phải là một số chính phương.

Bài 11. Tìm tất cả các số tự nhiên $n \geq 2$ sao cho tồn tại $n$ số nguyên liên tiếp mà tổng của chúng là một số chính phương.

Bài 12. Tìm $d$ sao cho với mọi $a,b \in {2,5,d}$ thì $ab-1$ là một số chính phương.

Bài 13. Chứng minh rằng với mọi $d$ thì tập ${2,5,13,d}$ luôn tồn tại hai số $a,b \in {2,5,13,d}$ sao cho $ab-1$ không phải là số chính phương.

Bài 14. Chứng minh rằng nếu tích của hai số nguyên tố cùng nhau là một số chính phương thì mỗi số cũng là số chính phương.

Bài 15. Cho các số nguyên dương $a, b$ thỏa $2{a^2} + a = 3{b^2} + b$.

a)Tìm $a, b$ biết $a$ và $b$ là hai số nguyên tố cùng nhau.
b) Chứng minh $a-b$ và $2a + 2b + 1$ là các số chính phương.

Bài 16. Cho các số nguyên $a, b, c$ thỏa $a + b + c$ chia hết cho 6 và ${a^2} + {b^2} + {c^2}$ chia hết cho 36. Đặt $A = {a^3} + {b^3} + {c^3}$

a) Chứng minh rằng A chia hết cho 8.
b) A có chia hết cho 27 không? Tại sao?

Bài 17. Cho $a,b,c$ là ba số nguyên dương thỏa $\dfrac{1}{a} – \dfrac{1}{b} = \dfrac{1}{c}$. Gọi $d$ là ước chung lớn nhất của ba số đó . Chứng minh rằng $d(b – a)$ là số chính phương.

 

Bài 18. Tìm tất cả các số nguyên dương $n$ sao cho $T = {2^n} + {3^n} + {4^n}$ là số chính phương.

 

Bài 19. Tìm tất cả các cặp số nguyên $a, b$ sao cho $3^a+ 7^b$ là một số chính phương.

Bài 20. (Chuyên Thái Bình 2021) Giả sử $n$ là số tự nhiên thỏa mãn điều kiện $n(n+1)+7$ không chia hết cho 7. Chứng minh rằng $4 n^{3}-5 n-1$ không là số chính phương.

Bài  21 (Thanh Hóa – Chuyên Tin 2021) Cho số tự nhiên $n \geqslant 2$ và số nguyên tố $p$ thỏa mãn $p-1$ chia hết cho $n$ và $n^{3}-1$ chia hết cho $p$. Chứng minh rằng $n+p$ là một số chính phương.

Bài 22 (Chuyên Lê Khiết) Cho các số nguyên tố $p, q$ thỏa mãn $p+q^{2}$ là số chính phương. Chứng minh rằng
a) $p=2 q+1$.
b) $p^{2}+q^{2021}$ không phải là số chính phương.

Bài 23 (Kiên Giang 2021) Cho $m, p, r$ là các số nguyên tố thỏa mãn $m p+1=r$. Chứng minh rằng $m^{2}+r$ hoặc $p^{2}+r$ là số chính phương.

Bài 24. (Chuyên Tiền Giang) Cho $m, n$ là các số nguyên dương sao cho $m^{2}+n^{2}+m$ chia hết cho $m n$. Chứng minh rằng $m$ là số chính phương.

Bài 25.(Chuyên Phổ thông Năng khiếu – ĐHQG thành phố Hồ Chí Minh 2021-2022)

a) Tìm tất cả số tự nhiên $n$ sao cho $(2 n+1)^{3}+1$ chia hết cho $2^{2021}$.
b) Cho số tự nhiên $n$ và số nguyên tố $p$ sao cho $a=\frac{2 n+2}{p}$ và $b=\frac{4 n^{2}+2 n+1}{p}$ là các số nguyên. Chứng minh rằng $a$ và $b$ không đồng thời là các số chính phương.

 

 

Phương pháp ánh xạ trong các bài toán tổ hợp

Thầy Nguyễn Tăng Vũ

1/ Lý thuyết:

Cho $A$ và $B$ là hai tập hợp hữu hạn.

  • Nếu có một đơn ánh $f: X \longrightarrow Y$ thì $|X| \le |Y|.$
  • Nếu có một toàn ánh $f: X \longrightarrow Y$ thì $|X| \ge |Y|.$
  • Nếu có một song ánh $f: X \longrightarrow Y$ thì $|X| = |Y|.$

2/ Bài tập:

Bài 1. 

Cho $X={ 1,2,..,n}$. Một tập con $S={s_1,s_2,…,s_k }$ của X ($s_1<s_2<…<s_k$) được gọi là \textit{m- tách được} $(m \in \mathbb{N})$ nếu $s_i-s_{i-1} \ge m; i=1,2,…,k$. Có bao nhiêu tập con m- tách được gồm $k$ phần tử của X, trong đó $0 \le k \le n-(m-1)(k-1)$.

Lời giải

Gọi A là tập tất cả các tập con m- tách được gồm $k$ phần tử của X và B là tập tất cả các tập con gồm k phần tử của tập $Y=\{1,2,…, n-(k-1)(m-1) \}$.

Ta xây dựng song ánh từ A đến B như sau: Với $S=\{s_1,s_2,…,s_k \} \in A$ ($s_1<s_2<…<s_k$) lấy tương ứng $f(S)=\{s_1, s_2-(m-1), s_3-2(m-1),…, s_k-(k-1)(m-1) \}$. Dễ chứng minh đây là một song ánh. Từ đó có $C^k_{n-(k-1)(m-1)}$ tập thoả yêu cầu đề bài.

Bài 2. 

Cho $X={1,2,…,n}$, với mỗi tập con khác rỗng $A_i={a_1,a_2,…,a_i }$ (không mất tổng quát giả sử $a_1>a_2>…>a_i$) ta định nghĩa tổng hỗn tạp của $A_i$ là số $m(A_i)=a_1-a_2+a_3-… \pm a_i$. Tính $\sum \limits_{A_i \subset X} m(A_i)$.

Lời giải

Gọi B là tập tất cả các tập con không chứa phần tử n của X và C là tập tất cả các tập con có chứa phần tử n của X.

Ta xây dựng song ánh từ B đến C như sau: Với $S=\{s_1,s_2,…,s_k \} \in B$ ($s_1<s_2<…<s_k<n$) lấy tương ứng $f(S)=\{s_1,s_2,…,s_k,n \} ,$ nhận thấy $f(S)\in C$. Dễ chứng minh đây là một song ánh.

Ta có: $|B|=|C|=2^{n-1};B\cup C= \emptyset;B\cap C=X$

Và $m(S)+m(f(S))=n$

Do đó: $$\sum \limits_{A_i \subset X} m(A_i) =\sum \limits_{B_i \subset B} m(B_i) +\sum \limits_{C_i \subset C} m(C_i)=n\cdot 2^{n-1}$$

Bài 3. 

Cho $X={1,2,…,n}$. Một tập con A của X được gọi là tập béo nếu mỗi phần tử của A đều không nhỏ hơn số phần tử của nó. Tập rỗng cũng là một tập béo. Đặt $a_n$ là số các tập béo của X mà trong mỗi tập không chứa hai số liên tiếp, $b_n$ là số các tập con của X mà hai phần tử bất kỳ hơn kém nhau ít nhất 3 đơn vị. Chứng minh $a_n=b_n.$

Lời giải

Gọi A là họ các tập béo thỏa yêu cầu đề bài, B là họ các các tập con của X có tính chất hai phần tử bất kỳ hơn kém nhau 3 đơn vị. Ta thiết lập một ánh xạ $f$ đi từ A đến B như sau: giả sử $x=\{a_1,a_2,…,a_k\} \in A$, ta có thể giả sử $k \le a_1<a_2<a_3<…<a_k \le n$. Đặt $b_1=a_1-k+1, b_2=a_2-k+2,…,b_k=a_k$. Khi đó $$ a_{i+1} \ge a_i+2, i=1,2,…,k-1. $$

Suy ra $a_{i+1}-a_i \ge 2$ do đó $b_{i+1}-b_i \ge 3$ và $b_1 \ge 1, b_k \le n.$ Định nghĩa $f(x)=y=\{b_1,b_2,…,b_k\}$, suy ra $y \in B$. Vậy $f$ là một ánh xạ, hơn nữa dễ thấy $f$ là một song ánh do đó ta có điều cần chứng minh.

Bài 4. 

Cho số nguyên dương $n$ và $d$ là một ước dương của $n$. Gọi S là tập tất cả những bộ $(x_1,x_2,…,x_n)$ nguyên dương thỏa $0 \le x_1 \le x_2 \le… \le x_n \le n$ và $d| x_1+x_2+…+x_n$. Chứng minh rằng có đúng một nửa các phần tử của S có tính chất $x_n=n$.

Lời giải

$(x_1,x_2,…,x_{n-1},x_n) \in S$ mà $x_n \ne n$ ta cho tương ứng với bộ $(x_1+1,x_2+2,…,x_{n-1}+1,x_n+1)$, với $(x_1,x_2,…,x_{n-1},n) \in S$ ta cho tương ứng với bộ $(0,x_1,…,x_{n-1}).$ \

Dễ chứng minh $f$ là một song ánh, $f$ hoán vị S và $f(S)=S.$ Vì tổng tất cả các phần tử của các bộ trong S và $f(S)$ là như nhau trong khi tương ứng thứ nhất tăng tăng tổng này lên $n$ đơn vị, tương ứng thứ hai giảm tổng này đi $n$ đơn vị nên số lần xuất hiện của hai tương ứng này là như nhau. Từ đó suy ra điều cần chứng minh

Bài 5. 

Gọi $a_n$ là số các xâu nhị phân độ dài $n$ không chứa ba bit 0, 1, 0 liên tiếp. Gọi $b_n$ là số các xâu nhị phân độ dài $n$ chứa bốn bit 0, 0, 1, 1 hoặc 1, 1, 0, 0 liên tiếp. Chứng minh rằng $b_{n+1}=2a_n$ với mọi số nguyên dương $n$.

Lời giải

Gọi $A_n, B_n$ lần lượt là tập các xâu nhị phân độ dài $n$ thỏa điều kiện thứ nhất và thứ hai. Với mỗi xâu nhị phân $(x_1,x_2,…,x_n)$ ta cho tương ứng với một xâu nhị phân $(y_0,y_1,…,y_n)$ xác định bởi $y_0=0$ và $$ y_i=x_1+x_2+…+x_i \ mod \ 2, i=1,2,…,n. \ \ \ (*)$$

Khi đó $$ x_i=y_i+y_{i-1} \ mod \ 2, i=1,2,…,n. $$

Dễ thấy (*) là một song ánh giữa tập tất cả các xâu nhị phân độ dài $n$ và tập tất cả các xâu nhị phân độ dài $n+1$ trong đó có bit đầu tiên là 0. Hơn nữa xâu nhị phân $(x_1,x_2,…,x_n)$ có 3 bit 0,1,0 liên tiếp theo thứ tự này khi và chỉ khi xâu nhị phân tương ứng $(y_0,y_1,…,y_n)$ có 4 bit liên tiếp theo thứ tự là 0,0,1,1 hoặc 1,1,0,0. Nói cách khác một xâu nhị phân thuộc $A_n$ sẽ tương ứng với một xâu nhị phân thuộc $B_{n+1}$ và bắt đầu bằng bit 0. Vì số xâu nhị phân thuộc vào $B_{n+1}$ bắt đầu bằng bit 0 đúng bằng một nửa số xâu nhị phân thuộc vào $B_{n+1}$ do đó ta có $b_{n+1}=2a_n$ (điều phải chứng minh).

 

Định lý Ceva và Menelaus – Phần 3

Phần 2

Ví dụ 10. (USAMO 2012) Gọi $P$ là một điểm thuộc miền trong tam giác $ABC$ và $d$ là một đường thẳng qua $P$. Đường thẳng đối xứng của $PA$ qua $d$ cắt $BC$ tại $A’$; các điểm $B’, C’$ được xác định tương tự. Chứng minh rằng $A’, B’, C’$ thẳng hàng.

Lời giải

Ta có $\dfrac{A’B}{A’C} = \dfrac{S_{A’PC}}{S_{A’PC}} = \dfrac{PB\cdot \sin A’PB}{PC\cdot\sin A’PC}$. (1)
Tương tự ta cũng có $\dfrac{B’C}{B’A} = \dfrac{PC \cdot \sin B’PC}{PA \cdot \sin B’PA}$ và $\dfrac{C’A}{C’B} = \dfrac{PA \cdot \sin C’PA}{PB \cdot \sin C’PB}$. (2)
Theo tính chất đối xứng ta có $\sin A’PB = \sin B’PA,\\ \sin A’PC = \sin C’PA, \sin B’PC = \sin C’PB$. (3)
Từ (1), (2), (3) ta có $$\dfrac{A’B}{A’C}\cdot \dfrac{B’C}{B’A}\cdot \dfrac{C’A}{C’B} = 1$$
Do đó $A’,B’,C’$ thẳng hàng.

Ví dụ 11. Cho tam giác $ABC$. Ba đường tròn $w_a, w_b, w_c$ lần lượt đi qua các cặp đỉnh $B,C$; $C, A$; và $A, B$. Gọi $D, E, F$ lần giao điểm thứ hai của ba đường tròn này. Đường thẳng qua $D$ vuông góc với $AD$ cắt $BC$ tại $X$; các điểm $Y, Z$ được xác định tương tự. Chứng minh rằng $X, Y, Z$ thẳng hàng.

Lời giải

Ta có $\dfrac{XB}{XC} = \dfrac{DB\sin XDB}{DC \sin XDC}$;
$\dfrac{DB}{DC} = \dfrac{R_c \sin DAB}{R_b \sin DAC}$ và $\dfrac{\sin ADB}{\sin XDC} = \dfrac{\cos ADB}{\cos ADC}$;
Tương tự cho các phân thức $\dfrac{YC}{YA}, \dfrac{ZA}{ZB}$.
Mặt khác ta có $AD, BE, CZ$ đồng quy tại tâm đẳng phương nên $\dfrac{\sin DAB}{\sin DAC}\cdot \dfrac{\sin EBC}{\sin EBA}\cdot \dfrac{\sin FCA}{\sin FCB} = 1$.
Từ đó ta có $\dfrac{XB}{XC} \cdot \dfrac{YC}{YA} \cdot \dfrac{ZA}{ZB}=1$.
Vậy $X, Y, Z$ thẳng hàng.

Ví dụ 12. (IMO shortlist 2013) Cho tam giác $ABC$ nhọn. Gọi $O$ là tâm ngoại tiếp và $H$ là trực tâm tam giác $ABC$. Chứng minh rằng tồn tại các điểm $D, E, F$ thuộc các cạnh $BC, AC, AB$ thỏa: $OD + DH = OE+EH = OF + FH$ và $AD, BE, CF$ đồng quy.

Lời giải

Gọi $H_1$ là điểm đối xứng của $H$ qua $BC$, thì $H_1 \in (O)$.
Gọi $D$ là giao điểm của $OH_1$ và $BC$, khi đó $OD + DH = OD + DH_1 = OH_1 = R$.
Các điểm $E, F$ được xác định tương tự ta có $OD + DH = EO +EH = OF + FH$.
Ta cần chứng minh $AD, BE, CF$ đồng quy bằng định lý Ceva dạng sin.
Ta có $\dfrac{DB}{DC} = \dfrac{S_{BH_1D}}{S_{CH_1D}} = \dfrac{BH_1.\sin BH_1D}{CH_1 \sin CH_1D} = \dfrac{BH}{CH}\dfrac{\sin B}{\sin C}$
Các đẳng thức kia tương tự, nhân lại ta có điều cần chứng minh.

Ví dụ 13. Cho tam giác $ABC$ khác tam giác cân nội tiếp đường tròn $w$, các đường trung tuyến từ $A, B,C$ cắt $w$ tại $A’, B’, C’$. Gọi $A_1$ là giao điểm của tiếp tuyến tại $A’$ với $BC$; các điểm $B_1, C_1$ được xác định tương tự. Chứng minh rằng $A_1, B_1, C_1$ thẳng hàng.

Lời giải

Ta có $A_1B\cdot A_1C = A_1A’^2 \Rightarrow \dfrac{A_1B}{A_1C} = \dfrac{A_1B^2}{A_1A’^2} = \dfrac{\sin^2 A_1A’B}{\sin^2 A_1BA’} = \dfrac{\sin^2 A’AB}{\sin^2 A’AC}$.
Chứng minh tương tự cho các đẳng thức kia và nhân lại, áp dụng ceva sin cho 3 đường $AA’, BB’, CC’$ đồng quy.

Bài tập rèn luyện

 

Bài 1. Cho tứ giác $ABCD$, gọi $I$ là giao điểm của $AC$ và $BD$, $K$ là giao điểm của $AB$ và $CD$. Đường thẳng $IK$ cắt các cạnh $BC$ và $AD$ tại $P, Q$.
Chứng minh rằng: $ \dfrac{\overline{IP}}{\overline{IQ}} = -\dfrac{\overline{KP}}{\overline{KQ}}$

Bài 2. Cho tứ giác $ABCD$ ngoại tiếp đường tròn $w$, $w$ tiếp xúc với các cạnh $AB, BC, CD, DA$ lần lượt tại $M, N, P, Q$. Chứng minh $MQ, BD, PN$ song song hoặc đồng quy.

Bài 3. Cho tam giác $ABC$, đường phân giác ngoài góc $A$ cắt đường thẳng vuông góc với $BC$ kẻ từ $B$ và $C$ lần lượt tại $D$ và $E$. Chứng minh rằng $BE, CD$ và $AO$ đồng quy, với $O$ là tâm đường tròn ngoại tiếp tam giác $ABC$.

Bài 4. Gọi $I$ là tâm đường tròn nội tiếp của tam giác $ABC$. Gọi $A’, B’, C’$ lần lượt là điểm đối xứng của $I$ qua $BC, AC, AB$. Chứng minh rằng $AA’, BB’, CC’$ đồng quy.

Bài 5. Cho tam giác $ABC$. Về phía ngoài tam giác dựng các hình vuông $BCDE, ACFG, ABHK$ với tâm lần lượt là $O_1, O_2, O_3$. Chứng minh $AO_1, BO_2, CO_3$ đồng quy.

Bài 6. Cho tam giác $ABC$ không cân tại $A$. $M$ là một điểm nằm trong tam giác thỏa $\angle AMB – \angle ACB = \angle AMC – \angle ABC$. Chứng minh rằng đường thẳng nối tâm đường tròn nội tiếp tam giác $AMB$ và $AMC$ đi qua một điểm cố định.

Bài 7. Cho tam giác $ABC$ và điểm $M$ nằm trong tam giác. $AM, BM, CM$ cắt $BC, AC, AB$ lần lượt tại $A’, B’, C’$. Gọi $P$ là giao điểm của $BB’$ và $A’C’$; $Q$ là giao điểm của $CC’$ và $A’B’$. Chứng minh rằng: $$\angle MAP = \angle MAQ \Leftrightarrow \angle MAB = \angle MAC$$

Bài 8. Cho tam giác $ABC$. Gọi $O$ là tâm đường tròn ngoại tiếp tam giác $ABC$; $O_1, O_2, O_3$ lần lượt là tâm ngoại tiếp các tam giác $BCO, ACO$ và $ABO$. Chứng minh rằng $AO_1, BO_2, CO_3$ đồng quy tại một điểm.(Điểm Kosnita)

Bài 9. Cho tam giác $ABC$ có $M$ là trung điểm cạnh $AB$. $CE$ là phân giác góc $\angle ACB$. $D$ thuộc tia đối của tia $CA$ sao cho $CD = CB$. Gọi $K$ là giao điểm của $DM$ và $CE$. Chứng minh rằng $\angle KBC = \angle BAC$.

Bài 10. Cho tam giác $ABC$ nhọn nội tiếp đường tròn $(O)$ và có trực tâm $H$. Gọi $A_o, B_o, C_o$ là trung điểm của $BC, AC, AB$. $A_1$ là giao điểm của $AA_o$ và $(O)$, $A_2$ là giao điểm của $H$ qua $A_o$; đường thẳng $A_1A_2$ cắt $BC$ tại điểm $S_a$; các điểm $S_b, S_c$ được xác định tương tự. Chứng minh $S_a, S_b, S_c$ thẳng hàng.

Bài 11. Cho tam giác $ABC$. Các điểm $A_1, B_1, C_1$ lần lượt thuộc các cạnh $BC, AC, AB$ sao cho các đường thẳng $AA_1, BB_1, CC_1$ đồng quy.

a) Gọi $A_2$ là điểm đối xứng của $A_1$ qua trung điểm cạnh $BC$; các điểm $B_2, C_2$ được xác định tương tự. Chứng minh rằng $AA_2, BB_2, CC_2$ cũng đồng quy.
b) Đường tròn ngoại tiếp tam giác $A_1B_1C_1$ cắt $BC, AC, AB$ tại $A_3, B_3, C_3$. Chứng minh $AA_3, BB_3, CC_3$ đồng quy.

 

Bài 12. Cho tam giác $ABC$. Các điểm $A_1, B_1, C_1$ lần lượt thuộc các cạnh $BC, AC$ và $AB$. Gọi $G_a, G_b, G_c$ lần lượt là trọng tâm các tam giác $AB_1C_1, BC_1A_1, CA_1B_1$. Chứng minh rằng $AG_a, BG_b, CG_c$ đồng quy khi và chỉ khi $AA_1, BB_1, CC_1$ đồng quy.

Bài 13.(IMO SL 1995) Đường tròn nội tiếp tam giác $ABC$ tiếp xúc với các cạnh $BC, AC, AB$ tại $D, E, F$. $X$ là điểm bên trong tam giác $ABC$ sao cho đường tròn nội tiếp tam giác $XBC$ tiếp xúc với $BC$ tại $D$, tiếp xúc với $CX, BX$ tại $Y, Z$. Chứng minh rằng $E, F, Z, Y$ cùng thuộc một đường tròn.

Bài 14. Cho $P$ là điểm thuộc miền trong của tam giác $ABC$. Gọi $D, E, F$ là hình chiếu của $P$ trên $BC, AC, AB$. Gọi $X$ là điểm trên $EF$ sao cho $PX \bot PA$; các điểm $Y, Z$ được xác định tương tự. Chứng minh rằng các điểm $X, Y, Z$ thẳng hàng.

Bài 15. (IMO SL 2006) Cho tam giác $ABC$ có $\angle ACB < \angle BAC < 90^o$.Lấy $D$ là điểm thuộc cạnh $AC$ sao cho $BD = BA$. Đường tròn nội tiếp tam giác $ABC$ tiếp xúc với $AB$ tại $K$ và $AC$ tại $L$. Gọi $J$ là tâm đường tròn nội tiếp tam giác $BCD$. Chứng minh rằng đường thẳng $KL$ chia đôi đoạn $AJ$.

Bài 18. Cho tam giác $ABC$ nội tiếp đường tròn tâm $O$. Gọi $A_1$ là điểm đối xứng của $A$ qua $O$, gọi $A_2$ là điểm đối xứng của $O$ qua $BC$; các điểm $B_1, B_2, C_1, C_2$ được xác định tương tự. Chứng minh rằng đường tròn ngoại các tam giác $OA_1A_2 OB_1B_2$ và $OC_1C_2$ cùng đi qua 2 điểm.

Bài 19. Cho tam giác $ABC$, đường tròn tâm $I$ nội tiếp tam giác và tiếp xúc với các cạnh $BC, AC, AB$ tại $D, E, F$. $X$ là điểm nằm trong tam giác $DEF$, gọi $A_1, A_2$ là giao điểm của $DX$ với $EF$ và $(I)$; các điểm $B_1,B_2$;$C_1,C_2$ được xác định tương tự.

a) Chứng minh $AA_2, BB_2, CC_2$ đồng quy tại $Y$; $AA_1, BB_1, CC_1$ đồng quy tạu $Z$.
b) Chứng minh $X, Y, Z$ thẳng hàng.

 

Bài 20. Cho một đường tròn với hai dây $AB$ và $CD$ không song song. Đường vuông góc với $AB$ kẻ từ $A$ cắt đường vuông góc với $CD$ kẻ từ $C$ và từ $D$ lần lượt tại $M, P$. Đường vuông góc với $AB$ kẻ từ $B$ cắt đường vuông góc với $CD$ kẻ từ $C$ và $D$ lần lượt tại $Q$ và $N$. Chứng minh rằng các đường thẳng $AD, BC, MN$ đồng quy và các đường thẳng $AC, BD, PQ$ cũng đồng quy.

Bài 21. (IMO shortlis 2011) Cho $ABC$ là một tam giác với đường tròn nội tiếp tâm $I$ và đường tròn ngoại tiếp $(C)$. $D$ và $E$ là giao điểm thứ hai của $(C)$ với các tia $AI$ và $BI$ tương ứng. $DE$ cắt $AC$ tại điểm $F$, và cắt $BC$ tại điểm $G$. $P$ là giao điểm của đường thẳng đi qua $F$ song song với $AD$ và đường thẳng qua $G$ song song với $BE$. Giả sử rằng $K$ là giao điểm của các tiếp tuyến của $(C)$ tại $A$ và $B$. Chứng minh rằng ba đường thẳng $AE, BD$ và $KP$ là song song hoặc đồng quy.

Bài 22. (China TST 2014) Cho tam giác $ABC$ nội tiếp đường tròn $(O)$; $H_a$ là chân đường cao hạ từ $A$ của tam giác $ABC$. $AO$ cắt đường tròn ngoại tiếp tam giác $BOC$ tại $A’$. Gọi $D, E$ là hình chiếu của $A’$ trên $AB$ và$AC$; và $O_a$ là tâm đường tròn ngoại tiếp tam giác $DEH_a$; Ta định nghĩa các điểm $H_b, O_b, H_c, O_c$ tương tự. Chứng minh rằng $H_aO_a, H_bO_b$ và $H_cO_c$ đồng quy.

 

Định lý Ceva và Menelaus – Phần 2

Trong hình học ta gặp nhiều bài toán về chứng minh ba đường đồng quy và ba điểm thẳng hàng, một trong những công cụ quen thuộc và kinh điển nhất là định lý Ceva và định lý Menelaus. Ngoài việc áp dụng chứng minh thẳng hàng đồng quy, các định lý Ceva và Nemelaus còn áp dụng chứng minh các đẳng thức về độ dài, góc, là cơ sở của những phương pháp mạnh khác như: hàng điểm điều hòa, cực đối cực,…

Hai định lý được phát biểu với dạng hình học, dạng đại số và dạng lượng giác, trong phần này ta ưu tiên các phát biểu dưới dạng độ dài hình học, góc hình học vì sự đơn giản của nó.

Định lý Ceva

(Dạng độ dài hình học) Cho tam giác $ABC$, nếu $A_1, B_1, C_1$ là là các điểm thuộc các cạnh $BC, AC, AB$. Khi đó $AA_1, BB_1, CC_1$ đồng quy khi và chỉ khi:

\begin{equation} \dfrac{A_1B}{A_1C} \cdot \dfrac{B_1C}{B_1A}\cdot \dfrac{C_1A}{C_1B} = 1
\end{equation}

(Dạng độ dài đại số) Cho tam giác $ABC$ và các điểm $A_1, B_1, C_1$ lần lượt thuộc các đường thẳng $BC, AC, AB$. Khi đó các đường thẳng $AA_1, BB_1, CC_1$ song song hoặc đồng quy khi và chỉ khi:
\begin{equation}\label{ceva2}
\dfrac{\overline{A_1B}}{\overline{A_1C}}.\dfrac{\overline{B_1C}}{\overline{B_1A}}.\dfrac{\overline{C_1A}}{\overline{C_1B}}=-1
\end{equation}

(Dạng lượng giác) Cho tam giác $ABC$ và các điểm $A_1, B_1, C_1$ lần lượt thuộc các đường thẳng $BC, AC, AB$. Khi đó các đường thẳng $AA_1, BB_1, CC_1$ song song hoặc đồng quy khi và chỉ khi:
\begin{equation}\label{ceva3}
\dfrac{\sin(AA_1;AB)}{\sin(AA_1;AC)}\cdot \dfrac{\sin(BB_1;BC)}{\sin(BB_1;BA)}\cdot \dfrac{\sin(CC_1;CA)}{\sin(CC_1;CB)}=-1
\end{equation}

Định lý Menelaus

(Dạng độ dài hình học) Cho tam giác $ABC$, các điểm $C_1$ thuộc cạnh $AB$; $B_1$ thuộc cạnh $AC$ và $A_1$ thuộc phần kéo dài của cạnh $BC$. Khi đó $A_1, B_1, C_1$ thẳng hàng khi và chỉ khi
\begin{equation}\label{mene1}
\dfrac{A_1B}{A_1C} \cdot \dfrac{B_1C}{B_1A}\cdot \dfrac{C_1A}{C_1B} = 1 \end{equation}

(Dạng độ dài đại số) Cho tam giác $ABC$ và các điểm $A_1, B_1, C_1$ lần lượt thuộc các đường thẳng $BC, AC, AB$. Khi đó các điểm $A_1, B_1, C_1$ thẳng hàng khi và chỉ khi:
\begin{equation}\label{mene2}
\dfrac{\overline{A_1B}}{\overline{A_1C}}\cdot \dfrac{\overline{B_1C}}{\overline{B_1A}}\cdot \dfrac{\overline{C_1A}}{\overline{C_1B}}= 1
\end{equation}

(Dạng lượng giác) Cho tam giác $ABC$ và các điểm $A_1, B_1, C_1$ lần lượt thuộc các đường thẳng $BC, AC, AB$. Khi đó các điểm $A_1, B_1, C_1$ thẳng hàng khi và chỉ khi:
\begin{equation}\label{mene3}
\dfrac{\sin(AA_1;AB)}{\sin(AA_1;AC)}\cdot \dfrac{\sin(BB_1;BC)}{\sin(BB_1;BA)}\cdot \dfrac{\sin(CC_1;CA)}{\sin(CC_1;CB)}=1
\end{equation}

Các ví dụ về định lý Ceva và Menelaus

Ví dụ 1. Cho tứ giác $ABC$, các đường chéo $AC, BD$ cắt nhau tại $I$; $AD, BC$ cắt nhau tại $E$; $AB, CD$ cắt nhau tại $F$. $EI$ cắt $AB, CD$ tại $K, L$. Khi đó $\dfrac{LC}{LD} = \dfrac{FC}{FD}$.

Lời giải

Áp dụng định lý Ceva cho tam giác $ECD$ ta có $$\dfrac{LD}{LC} \cdot \dfrac{BC}{BA} \cdot \dfrac{AE}{AD} = 1$$
Áp dụng Menelaus cho cho tam giác $ECD$ với 3 điểm $F, A, B$ ta có: $$\dfrac{FD}{FC}\cdot \dfrac{BC}{BA} \cdot \dfrac{AE}{AD} = 1$$
Từ trên ta có $\dfrac{LD}{LC} = \dfrac{FD}{FC}$.

Ví dụ 2. (Đường thẳng Gauss) Cho tứ giác $ABCD$ khác hình thang. Gọi $I$ là giao điểm của $AD, BC$; gọi $J$ là giao điểm của $AB, CD$. Chứng minh rằng trung điểm của các đoạn $AC, BD$ và $IJ$ cùng thuộc một đường thẳng.

Lời giải

Gọi $E, F, H$ lần lượt là trung điểm của $AD, IC, CD$. \\Rõ ràng $P \in EF, M \in FH, N \in EH$. \\
Ta có $\dfrac{PE}{PF} = \dfrac{JD}{JC}$; $\dfrac{NH}{NE} = \dfrac{BC}{BI}$ và $\dfrac{MF}{MH} = \dfrac{AI}{AD}$.\hfill (1)\\
Áp dụng Menelaus cho tam giác $IDC$ với 3 điểm thẳng hàng $J, A, B$ ta có: \\
$\dfrac{JD}{JC}\cdot \dfrac{BC}{BI}\cdot \dfrac{AI}{AD} = 1$. \hfill (2)\\
Từ (1) và (2) suy ra $\dfrac{PE}{PF}\cdot \dfrac{JD}{JC}\cdot \dfrac{MF}{MH}= 1$.\\ Do đó 3 điểm $P, N, M$ thẳng hàng.

Ví dụ 3. Cho tứ giác $ABCD$, trên các cạnh $AD, BC$ lấy các điểm $P, Q$ sao cho $\dfrac{AP}{AD} = \dfrac{BQ}{BC}$. Gọi $I$ là giao điểm $AC, BD$ và $K$ là giao điểm của $DQ, CP$. Chứng minh $PQ$ đi song song với đường thẳng qua trung điểm của $AB, CD$.

Lời giải

Gọi $E$ là giao điểm của $AD, BC$; $X, Y$ lần lượt là trung điểm của $IE$ và $PQ$; $M, N$ là trung điểm $AB, CD$. \\
Theo định lý đường thẳng Gauss ta có $M, N, X$ thẳng hàng. \\
Mặt khác do $\dfrac{AP}{AD} = \dfrac{BQ}{BC}$ nên $Y, M, N$ thẳng hàng. Do đó 4 điểm $X, M, N, Y$ thẳng hàng.\\
Theo định lý Thales ta có $XM \parallel IK$.\\
Từ đó ta có $IK \parallel MN$.

Ví dụ 4. Cho tam giác $ABC$ ngoại tiếp đường tròn $w$ tâm $I$, $w$ tiếp xúc với các cạnh $BC, AC, AB$ lần lượt tại $D, E, F$.

a) Chứng minh các đường thẳng $AD, BE$ và $CF$ đồng quy tại một điểm. (Điểm Gergonne)
b) Gọi $D’, E’, F’$ lần lượt là điểm đối xứng của $D, E, F$ qua $I$. Chứng minh rằng $AD’, BE’, CF’$ đồng quy tại một điểm.(Điểm Nagel)

Lời giải

a)Ta có $BD = BF, CD = CE, AE = AF$. Suy ra $\dfrac{BD}{CD}\cdot \dfrac{CE}{AE}\cdot \dfrac{AE}{AF} = 1$. Do đó $AD, BE, CF$ đồng quy.
b) Cho $AD’$ cắt $BC$ tại $D_1$; các điểm $E_1, F_1$ được xác định tương tự. \\
Vẽ đường thẳng qua $D’$ song song với $BC$ cắt $AB, AC$ tại $L,K$. Ta có $D’K\cdot CD = KE\cdot CE = IE^2$; $D’L\cdot BD = LF\cdot BF = ID^2$.\\
Suy ra $D’K\cdot CD = D’L\cdot BD$, suy ra $\dfrac{D’K}{D’L} =\dfrac{DB}{CD}$.\\
Mặt khác $\dfrac{D’K}{CD_1} = \dfrac{AD’}{AD_1} = \dfrac{D’L}{BD_1}$, suy ra $\dfrac{D’K}{D’L} = \dfrac{CD_1}{BD_1}$.\\
Do đó $\dfrac{BD}{CD} = \dfrac{CD_1}{CD_1}$, suy ra $BD = CD_1$.\\
Chứng minh tương tự ta có $CE = AE_1, BF = AF_1$.
Từ đó ta có các đường thẳng $AA_1, BB_1, CC_1$ đồng quy.

Ví dụ 5. Cho tam giác $ABC$ nội tiếp đường tròn $w$. Tiếp tuyến tại $A$ của $w$ cắt $BC$ tại $A’$; các điểm $B’, C’$ được xác định tương tự. Chứng minh rằng $A’, B’, C’$ thẳng hàng.

Lời giải

Mà $\triangle A’AB \backsim \triangle A’CA$ nên $\dfrac{A’A^2}{A’C^2} = \dfrac{AB^2}{AC^2}$. \\
Chứng minh tương tự ta có: $\dfrac{B’C}{B’A}= \dfrac{BC^2}{AB^2}, \dfrac{C’A}{C’B} = \dfrac{AC^2}{BC^2}$.\\
Khi đó $\dfrac{A’B}{A’C}\cdot \dfrac{B’C}{B’A}\cdot \dfrac{C’A}{C’B} = 1$.
Vậy $A’, B’, C’$ thẳng hàng.

Ví dụ 6. Cho tam giác $ABC$ khác tam giác cân. Đường tròn tâm $I$ nội tiếp tam giác và tiếp xúc với các cạnh $BC, AC, AB$ lần lượt tại $D, E, F$. Chứng minh rằng tâm đường tròn ngoại tiếp các tam giác $ADI, BEI, CFI$ thẳng hàng.

Lời giải

Gọi $D’$ là chân đường phân giác ngoài của góc $A$, khi đó $I, A, D, D’$ thuộc đường tròn đường kính $ID’$, suy ra tâm $O_1$ của $(IDA)$ là trung điểm của $ID’$. \\
Xác định tương tự cho $E’, F’$. Ta có tâm của $(IBE), (ICF)$ lần lượt là trung điểm của $IE’, IF$. \\
Sử dụng Menelaus ta chứng minh được $D’, E’, F’$ thẳng hàng.
Do đó $O_1, O_2,O_3$ thẳng hàng.

Ví dụ 7. (Định lý Jacobi) Cho tam giác $ABC$. Về phía ngoài tam giác lấy các điểm $D, E, F$ sao cho $\angle DBC = \angle FBA, \angle DCB = \angle ECA, \angle EAC = \angle FAB$. Chứng minh rằng các đường thẳng $AD, BE$ và $CF$ đồng quy.

Lời giải

Để chứng minh định lý này, ta sử dụng định lý Ceva dạng sin, ta cần chứng minh $$\dfrac{\sin DAB}{\sin DAC}\cdot \dfrac{\sin EBC}{\sin EBA} \cdot \dfrac{\sin FCA}{\sin FCB} = 1$$
Áp dụng định lý Cevasin cho 3 đường đồng quy $AD, BD, CD$ ta có:
\begin{equation}
\dfrac{\sin DAB }{\sin DAC}\cdot \dfrac{\sin DBC}{\sin DBA}\cdot \dfrac{\sin DCA}{\sin DCB}
\end{equation}
Tương tự ta cũng có \begin{equation}
\dfrac{\sin EBC}{\sin EBA}\cdot \dfrac{\sin ECA}{\sin ECB}\cdot \dfrac{EAB}{\sin EAC} = 1 \end{equation} và
\begin{equation}
\dfrac{\sin FCA}{\sin FCB}\cdot \dfrac{FAB}{\sin FAC}\cdot \dfrac{FBC}{\sin FBA} = 1
\end{equation}
Nhân 3 đẳng thức lại và kết hợp $\angle DBC = \angle DBA, \angle DBA = \angle FBC, \angle DCB = \angle EDA \\ \angle DCA = \angle ECB, \angle FAB = \angle EAC, \angle FAC = \angle EAB$.
Ta có \begin{equation}
\dfrac{\sin DAB}{\sin DAC}\cdot \dfrac{\sin EBC}{\sin EBA }\cdot \dfrac{\sin FCA}{\sin FCB} =1
\end{equation}
Do đó $AD, BE, CF$ đồng quy.

Ví dụ 8. (Cevian Nest) Cho các đường thẳng $AX, BY, CZ$ đồng quy của tam giác $ABC$. Giả sử $XD, YE, CF$ là các đường đồng quy của tam giác $XYZ$. Chứng minh rằng $AD, BE, CF$ đồng quy.

Lời giải

Ví dụ 9. Cho tam giác $ABC$ nhọn. Về phía ngoài tam giác dựng các tam giác $ABD, ACE$ vuông tại $B, C$ và đồng dạng. Chứng minh rằng giao điểm của $BE$ và $CD$ thuộc đường cao hạ từ $A$ của tam giác $ABC$.

Lời giải

Áp dụng định lý Ceva sin cho các đường thẳng $BE, AE, CE$ ta có:
\begin{equation}
\dfrac{\sin EBC}{\sin EBA}\cdot \dfrac{\sin ECA}{\sin ECB}\cdot \dfrac{\sin EAB}{\sin EAC} = 1
\end{equation}

Tương tự ta có
\begin{equation}
\dfrac{\sin DCA}{\sin DCB}\cdot \dfrac{\sin DAB}{\sin DAC}\cdot \dfrac{\sin DBC}{\sin DBA} = 1
\end{equation}

Vẽ $AH \bot BC$, ta có $\sin BAH = \sin DBC, \sin CAH = \angle ECB$.\\
Hơn nữa $\angle EAB = \angle DAC, \angle ECA = \angle DBA = 90^\circ$. (3)\\
Nhân (1) và (2) kết hợp với 3 ta có:
\begin{equation}
\dfrac{\sin BAH}{\sin CAH}\cdot \dfrac{\sin EBC}{\sin EBA}\cdot \dfrac{\sin DCA}{\sin DCB} = 1
\end{equation}
Vậy $AH, BE, CD$ đồng quy.

Đếm bằng 2 cách

NGUYỄN QUANG TÂN

1/ Bài tập đếm số cặp

Bài 1. 

Trong một ủy ban, mỗi thành viên thuộc đúng 3 tiểu ban, mỗi tiểu ban có đúng 3 thành viên. Chứng minh rằng số thành viên của ủy ban bằng số tiểu ban.

Lời giải

Gọi số thành viên là $n$ và số tiểu ban là $m$.

Gọi $T$ là số cặp $(a, B)$ trong đó thành viên $a$ thuộc tiểu $B$. Vì mỗi thành viên thuộc 3 tiểu ban nên: $T=3 n$;

Vi mỗi tiểu ban có đúng 3 thành viên nên $T=3 m$;

Vậy $3 m=3 n$ nên $m=n$

Bài 2. 

Trong trường học có một số câu lạc bộ. Biết rằng mỗi câu lạc bộ có đúng 30 học sinh. Vâ mỗi học sinh tham gia đúng 3 câu lạc bộ. Biết rằng tổng số câu lạc bộ và số học sinh bằng 440 . Tìm số học sinh của trường.

Lời giải

Ta đếm số cặp $(a, B)$ mà học sinh $a$ thuộc câu lạc bộ $B$. Gọi $m$ là số câu lạc bộ và $n$ là số học sinh.

Ta có hệ $30 m=3 n$ và $m+n=440$. Ta được $m=40$ và $n=400$.

Bài 3. (IMC 2002)

Có 200 thí sinh tham gia một cuộc thi toán học. Các thí sinh phải giải 6 bài toán. Biết mỗi bài được giải đúng bởi ít nhất 120 thí sinh. Chứng minh rằng có hai thí sinh mà mỗi bài tập được giải bởi ít nhất một trong hai thí sinh.

Lời giải

Ta dùng phản chứng: Với mỗi cặp thí sinh bất kì đều có ít nhất 1 bài không giải được bởi cả hai thí sinh.

Gọi $T$ là số bộ $\left(\left\{t_{1}, t_{2}\right\} ; B\right)$ thỏa mãn thí $\sinh t_{1}, t_{2}$ cùng không giải được bài tập B.

Ta biết rằng mỗi bài được giải bởi ít nhất 120 thí sinh, vậy số thí sinh không giải được một bài nhỏ hơn hoặc bằng 80.

Nên $T \leq 6 C_{80}^{2}=18960$

Mặt khác hai thí sinh bất kì có ít nhất một bài cùng không giải được nên $T \geq C_{200}^{2}=19900 .$ Mâu thuẫn.

Vậy điều phản chứng là sai. Bài toán được chứng minh.

Bài 4. (Chọn đội tuyển thi Quốc Gia Khối chuyên ĐHSP 2013)

Cơ sở dữ liệu tạp chí của thư viện Quốc Gia có đúng 2016 loại khác nhau. Thư viện này cho phép 2013 thư viện địa phương kết nối để có thể khai thác cơ sở dữ liệu tạp chí của nó. Biết mỗi thư viện địa phương được phép khai thác ít nhất 1008 loại tạp chí khác nhau và 2 thư viện địa phương bất kì có tối đa 504 loại tạp chí mà cả 2 thư viện địa phương đó cùng đc phép khai thác. Chứng minh rằng không có quá 1 loại tạp chí trong cơ sở dữ liệu của thư viện Quốc Gia mà cả 2013 thư viện địa phương đều không thể khai thác được.

Lời giải

Giả sử có 2 tạp chí mà không thư viện địa phương nào cùng khai thác. Như vậy tập hợp các tạp chí mà các thư viện cùng khai thác là $X=$ $\{1, \ldots, 2014\} .$ Gọi $T$ là số bộ $\left(\left\{T_{1}, T_{2}\right\}, t\right)$ mà hai thư viện $T_{1}, T_{2}$ cùng khai thác tạp chí $t$.

Vì mỗi thư viện khai thác chung tối đa 504 tạp chí nên: $T \leq 504 C_{2013}^{2}$.

Giả sử số thư viện cùng khai thác tạp chí thứ $i$ là $a_{i}$ thi $\sum_{i=1}^{2013} a_{i} \geq 2013.1008$.

Ta có: $T=\sum_{i=1}^{2014} C_{a_{i}}^{2} \geq \frac{2003.1008(2003.1008-2014)}{2.2014}$

Suy ra: $504 \mathrm{C}_{2013}^{2} \geq \frac{2003.1008(2003.1008-2014)}{2.2014}$. Điều này vô lý.

Bài 5. (Chọn đội tuyển HSG QG tỉnh Đồng Tháp năm học 2013)

CLB du khảo có n thành viên. Năm ngoái CLB đã tổ chức được 6 chuyến du khảo, mỗi chuyến có 5 thành viên tham dự. Một thành viên CLB nhận xét rằng hai chuyến du khảo bất kỳ có không quá hai thành viên chung. Hỏi CLB đó có ít nhất bao nhiêu thành viên?

Lời giải

Giả sử câu lạc bộ đó có $n$ thành viên. Tập hợp các học sinh tham gia chuyến du khảo thứ $i$ là $A_{i}$. Ta có:

i) $\left|A_{i}\right|=5$;

ii) $\left|A_{i} \cap A_{j}\right| \leq 2$

Giả sử thành viên thứ $i$ tham gia $a_{i}$ chuyến du khảo ta có $\sum_{i=1}^{6} a_{i}=30$.

Gọi $T$ là số bộ $\left(A_{i}, A_{j}, k\right)$ trong đó thành viên $k$ tham gia cả hai chuyến du khảo $A_{i}$ và $A_{j}$

$$

T=\sum_{i=1}^{n} C_{a_{i}}^{2} \geq \frac{15(30-n)}{n}

$$

Vì hai chuyến du khảo bất kì có không quá 2 thành viên chung nên: $T \leq 2 C_{6}^{2}=30$

Suy ra: $\frac{15(30-n)}{n} \leq 30 \Leftrightarrow n \geq 10$

2/ Bài tập đếm số bộ ba

Bài 6.

Trong một trương học có $m$ lớp với $m \geq 10$, tổng số học sinh là $n$. Với $n \leq 465$, các lớp được đánh số từ 1 tới $m$, biết rằng mỗi học sinh ở lớp $i$ quen $j$ học sinh ở lớp thứ $j$. Tìm giá trị lớn nhất của $n$.

Lời giải

Giả sử lớp thứ $i$ có $k_{i}$ học sinh. Từ quan hệ quen biết ta có: $k_{i} \cdot j=k_{j} \cdot i$ nên $\frac{k_{i}}{i}=t$ với mọi $i$.

Vi $k_{1}+\cdots+k_{m}=n \leq 465$ nên $t(1+\cdots+m) \leq 465$ mà $m \geq 10$ nên $t \frac{10.11}{2}=55$

Suy ra $55 t \leq 465$ nên $n \leq 8$. Vậy $n$ lớn nhất bằng 8 .

Bài 7. (Hong Kong TST 2003)

Có 15 học sinh tham gia một khóa học. Biết rằng mỗi ngày, có 3 học sinh phải trực vệ sinh cho lớp và sau khi kết thúc khóa học thì cứ 2 học sinh bất kì đều tham gia trực chung đúng 1 lần. Tính số ngày khóa học diễn ra.

Lời giải

Giả sử khóa học có $m$ ngày. Ta gọi $S$ là số bộ $(x, y, z)$ trong đó hai học $\sinh x, y$ cùng tham gia trực nhật vào ngày thứ $z$ của khóa học.

Vì 2 học sinh bất kỳ đều tham gia trực nhật chung đúng một lần nên $S=$ $C_{15}^{2}=105$. Mặt khác mỗi ngày có 3 học sinh tham gia trực nhật nên $S=$ $m C_{3}^{2}=3 m$

Vậy $m=\frac{105}{3}=35$.

Bài 8. (IMO 1989)

Cho $n$ và $k$ là các số nguyên dương mà tồn tại một tập $T$ có $n$ điểm trong mặt phẳng sao cho

i) Không có 3 điểm nào thẳng hàng;

ii) $\forall P \in T$ có ít nhất $k$ điểm thuộc $T$ mà cách đều $P$.

Chứng minh rằng $k \leq \frac{1}{2}+\sqrt{2 n}$

Lời giải

Gọi $S$ là số các bộ 3 điểm $(\{A, B\}, C)$ mà $A, B$ cách đều $C$.

Trước hết ta đếm theo $\{A, B\}$ : Với mỗi cạ̣p điểm $\{A, B\}$ thì các điểm cách đều $A$ và $B$ nằm trên trung trực của đoạn $A B$. Mà không có 3 điểm nào thẳng hàng trong $T$ nên ta có chỉ có tối đa 2 điểm $C$. Vậy $S \leq 2 C_{n}^{2}$.

Ta đếm theo $C$ : Với mỗi điểm $C$ có ít nhất $k$ điểm trong $T$ cách đều $C$ nên số cặp $\{A, B\}$ ít nhất là $C_{k}^{2}$. Do đó $S \geq n C_{k}^{2}$.

Suy ra $2 C_{n}^{2} \geq n C_{k}^{2} .$ Từ đó ta có: $k \leq \frac{1}{2}+\sqrt{2 n}$

Bài 9.

Cho $X$ là một tập hữu hạn với $|X|=n$ và $A_{1}, A_{2}, \ldots, A_{m}$ là các tập con của $X$ có đúng 3 phần tử thỏa mãn $\left|A_{i} \cap A_{j}\right| \leq 1 \forall i \neq j$. Chứng minh rằng tồn tại một tập con $A \subset X$ với ít nhất $[\sqrt{2 n}]$ phần tử mà không chứa bất kì tập $A_{i}(1 \leq i \leq m)$ nào.

Lời giải

Giả sử $A \subset X$ là một tập con với số phần tử lớn nhất là $k$ mà không chứa bất kì tập $A_{i}(1 \leq i \leq m)$ nào.

Gọi $S$ là số bộ $(\{a, b\}, c)$ thỏa mãn $c \notin A$ và $\{a, b\}$ là tập con của một trong một tập $A_{1}, \ldots, A_{m}$.

Ta đếm theo $c$ : Với mỗi $c \notin A$ do tính cực đại của $A$ thì sẽ tồn tại $A_{i} \subset$ $A \cup\{c\}$ khi đó chọn $\{a, b\}=A_{i} \backslash\{c\} .$

Do vậy $S \geq n-k$.

Ta đếm theo $\{a, b\}:$ Do $\left|A_{i} \cap A_{j}\right| \leq 1 \forall i \neq j$ nên với mỗi $\{a, b\} \subset A$ tồn tại nhiều nhất một $A_{j} \supset\{a, b\}$, khi đó ta chọn $c=A_{j} \backslash\{a, b\} .$ Do đó $S \leq C_{k}^{2}$.

Suy ra $n-k \geq C_{k}^{2}$ hay $k^{2}+k \geq 2 n$ nên $k \geq-\frac{1}{2}+\frac{1}{2} \sqrt{1+8 n}$

Từ đó ta có: $k \geq\lfloor\sqrt{2 n}\rfloor$.

Bài 10. (APMO 2006) 

Trong một rạp xiếc, có $n$ chú hề được trang điểm bằng một số trong tổng cộng 12 màu sơn. Biết rằng mỗi chú hề phải sử dụng ít nhất 5 màu. Ngoài ra, mỗi màu được sử dụng bởi không quá 20 chú hề. Chứng minh rằng $n \leq 48$

Lời giải

Gọi số chú hề sử dụng màu sơn thứ $i$ là $a_{i} \leq 20 .$ Ta có $T=\sum_{i=1}^{12} a_{i} \geq$ 5n. Gọi $S$ là số bộ $(x, y, m)$ trong đó hai chú hề $x, y$ cùng sử dụng màu sơn thứ $m$

Ta có $S=\sum_{i=1}^{12} C_{a_{i}}^{2} \geq \frac{T(T-12)}{2.12} \geq \frac{5 n(5 n-12)}{24}$

Vì mỗi màu sơn được sử dụng bởi không quá 20 chú hề, $S \leq 12 C_{20}^{2}=2280$. Ta có phương trình $\frac{5 n(5 n-12)}{24} \leq 2280 \Rightarrow n \leq 48$

Bài 11. (China MO 1996)

Có 8 ca sĩ tham gia một chương trình văn nghệ với tổng cộng $m$ buổi hòa nhạc. Trong mỗi buổi hòa nhạc, có 4 ca sĩ tham gia và số lần tham gia của mỗi cặp ca sĩ là như nhau và bằng $n$.
Tìm giá trị nhỏ nhất của $m$.

Lời giải

Ta gọi $S$ là số bộ $(A, b, c)$ trong đó ca sĩ $b, c$ cùng tham gia buổi hòa nhạc $A .$

Vi hai ca sĩ bất kì cùng tham gia biểu diễn $n$ buổi nên $S=n C_{8}^{2}$.

Mặt khác có $m$ buổi biểu diễn và mỗi buổi có đúng 4 ca sĩ tham gia nên $S=m C_{4}^{2}$

Do vậy $n C_{8}^{2}=m C_{4}^{2}$ hay $14 n=3 m .$ Suy ra $14 \mid m$ nên $m \geq 14$. Giá trị nhỏ nhất của $m$ là 14 .

Dưới đây là một sách sắp xếp như vậy

Không có mô tả.

Bài 12. (VMO 2005)

Cho bát giác lồi $A_{1} A_{2} \ldots A_{8}$ mà không có ba đường chéo nào đồng quy. Giao của hai đường chéo tùy ý được gọi là một “nút”. Xét tất cả các tứ giác lồi được tạo thành bởi bốn đỉnh của bát giác đã cho và các tứ giác đó được gọi là “tứ giác con”. Hãy xác định số nguyên dương $n$ nhỏ nhất sao cho có thể tô màu $n$ nút để với mọi $i \neq k$ và $i, k \in{1,2, \ldots, 8}$ thì các số $s(i, k)$ bằng nhau, trong đó $s(i, k)$ ký hiệu số tứ giác con nhận $A_{i}, A_{k}$ làm đỉnh và giao của hai đường chéo là một nút được tô màu.

Lời giải

Bài toán này chính là bài toán China MO 1996. Thật vậy nếu ta coi các đỉnh của đa giác là các ca sĩ. Mỗi một nút được tô màu là là một buổi biểu diễn. Khi đó số $s(i, k)$ chính là số buổi biểu diễn mà ca sĩ $A_{i}, A_{k}$ cùng tham gia.

Bài 13. (Chọn đội tuyển PTNK 2014)

Cho tập hợp $X={1,2,3, \ldots, 19}$ và xét một họ $\Omega$ gồm $k$ tập con có 7 phần tử của $X$. Một tập hợp $A \subset X$ được gọi là “tập mẹ” của họ $\Omega$ nếu như $A$ có 8 phần tử và tồn tại một tập hợp $B \in \Omega$ sao cho $B \subset A .$ Gọi $d$ là số tất cả các tập mẹ của $\Omega .$ Chứng minh rằng $d \geq \frac{3}{2} k$.

Lời giải

Gọi $S$ là số bộ $(B, A)$ sao cho $B$ thuộc họ $\Omega$ và $A$ là mẹ của $B$.

Vì một tập $B$ thì có 7 phần tử, vậy $A$ muốn là mẹ của $B$ thì $A$ phải bổ sung thêm một trong 12 phần tử còn lại. Tức là mỗi tập $B$ sẽ có 12 tập mẹ. $S=k .12$ Mỗi tập $A$ có 8 tập con có 7 phần tử nên mỗi tập mẹ $A$ có nhiều nhất 8 tập $\operatorname{con} B .$ Ta có $S \leq 8 d$

Suy ra: $12 k \leq 8 d$. Vậy $d \geq \frac{3}{2} k$

Bài 14. (Russia 1996)

Ở một thành phố nọ, có 1600 đại biểu tham gia vào 16000 ủy ban và mỗi ủy ban có đúng 80 người tham gia. Chứng minh rằng có 2 ủy ban nào đó có ít nhất 4 đại biểu tham gia chung.

Lời giải

Gọi $a_{i}$ là số ủy ban mà đại biểu thứ $i$ tham gia. Ta gọi $T$ là số bộ $(x, Y)$ mà đại biểu $x$ tham gia ủy ban $Y$.

Trước hết ta đếm theo $Y$ thì $T=16000.80$.

Bây giờ ta đếm theo $x$ thì $T=\sum_{i=1}^{1600} a_{i}$.

Suy ra $\sum_{i=1}^{1600} a_{i}=16000.80$

Ta chứng minh bằng phản chứng, giả sử 2 ủy ban bất kì có không quá 3 đại biểu chung.

Ta gọi $S$ là số bộ $(x, V, U)$ trong đó đại biểu $x$ cùng tham gia cả 2 ủy ban $U$ và $V$. Ta có $S \leq 3 C_{16000}^{2}$

Bây giờ ta đếm theo $x$ thì $S=\sum_{i=1}^{1600} C_{a_{i}}^{2} \geq \frac{T(T-1600)}{2.1600}$.

Từ đó ta rút ra điều vô lý.

Bài 15. (Bulgari MO 2006)

Một quốc gia có 16 thành phố và có 36 tuyến bay nối giữa chúng (chuyến bay ở đây là hai chiều). Chứng minh rằng ta có thể tổ chức một chuyến bay vòng quanh giữa 4 thành phố nào đó.

Lời giải

Ta chứng minh bằng phản chứng. Giả sử với 2 sân bay $A, B$ bất kì có không quá một thành phố $C$ có thể bay đến cả $A$ và $B$.

Gọi $T$ là số bộ $(A, B, C)$ mà cả $A, B$ cùng bay được đến $C$. Ta có $T \leq C_{16}^{2}=120$

Giả sử thành phố thứ $i$ có thể bay đến được $a_{i}$ thành phố khác. Thì $\sum_{i=1}^{16} a_{i}=$ $72 .$

Khi đó: $\sum_{i=1}^{16} C_{a_{i}}^{2} \geq \frac{72(72-16)}{2.16}>120$. Vô lý.

Bài 16. (Chọn đội tuyển Bình Thuận)

Trong một hội nghị có 155 đại biểu tham dự và có 2015 cặp đại biểu quen biết nhau. Chứng minh rằng có thể chọn ra 4 đại biểu để xếp lên một bàn tròn sao cho hai đại biểu ngồi cạnh nhau thì có quen biết nhau.

Lời giải

Bài này cách giải hoàn toán giống bài Bulgari MO $2006 .$

Bài 17. 

Trong một trường có 1001 học sinh tham gia vào một số câu lạc bộ, các câu lạc bộ hợp với nhau để tổ chức $m$ hoạt động xã hội (một hoạt động có thể được tổ chức bởi một hoặc nhiều câu lạc bộ). Biết rằng
i) Mỗi câu lạc bộ đều có lẻ thành viên và nếu câu lạc bộ có $k$ thành viên thì nó tổ chức $\frac{k-1}{2}$ hoạt động xã hội;
ii) Với mỗi học sinh bất kì và một hoạt động bất kì thì có đúng một câu lạc bộ vừa tổ chức hoạt động xã hội đó và nhận học sinh đó là thành viên;
iii) Hai học sinh bất kì tham gia đúng một câu lạc bộ;

Tìm $m$.

Lời giải

Gọi $S$ là số bộ $(t, C, H)$ mà học sinh $t$ tham gia câu lạc bộ $C$ và câu lạc bộ $C$ tổ chức hoạt động $H$. Vì với mỗi cặp $t$ và $H$ chỉ tồn tại duy nhất một $C$ nên $S=1001 \mathrm{~m}$

Nếu một câu lạc bộ có $k$ thành viên thì nó tổ chức $\frac{k-1}{2}$. Giả sử có $n$ câu lạc bộ và câu lạc bộ thứ $i$ có $k_{i}$ thành viên thì $S=\sum_{i=1}^{n} k_{i} \frac{k_{i}-1}{2}$.
Suy ra $\sum_{i=1}^{n} \frac{k_{i}^{2}-k_{i}}{2}=1001 \mathrm{~m}$.
Gọi $T$ là số bộ $(a, b, C)$ là hai học sinh $a, b$ cùng tham gia câu lạc bộ $C$. Vi hai học sinh bất kì chỉ tham gia đúng một câu lạc bộ nên $T=C_{1001}^{2}$.
Mặt khác câu lạc bộ thứ $i$ có $k_{i}$ thành viên nên $T=\sum_{i=1}^{n} C_{k_{i}}^{2}$.
Suy ra $\sum_{i=1}^{n} \frac{k_{i}^{2}-k_{i}}{2}=C_{1001}^{2}$
Vậy $10001 m=C_{10001}^{2}$ nên $m=500$.

Bài 18. 

Một trường có $n$ học sinh và $m$ câu lạc bộ. Mỗi câu lạc bộ có ít nhất 2 thành viên. Nếu 2 câu lạc bộ có nhiều hơn một thành viên chung thì sô thành viên của 2 câu lạc bộ khác nhau.

a) Gọi $a_{i}$ là số câu lạc bộ có $i$ thành viên. Chứng minh $a_{i} \leq \frac{C_{n}^{2}}{C_{i}^{2}}$;

b) Chứng minh $m \leq n-1$;

Lời giải

a) Ta gọi $S$ là số bộ $(x, y, C)$ mà hai học sinh $x, y$ cùng tham gia CLB C có $i$ thành viên. Trước hết ta đếm theo $x$ và $y$. Vi 2 câu lạc bộ có cùng hai thànhviên $x, y$ chung thì số thành viên của 2 câu lạc bộ khác nhau nên có nhiều nhất một câu lạc bộ có $i$ thành viên và chứa $x, y$.
Suy ra $S \leq C_{n}^{2}$
Bây giờ ta đếm theo $\mathrm{C}$, số câu lạc bộ có $i$ thành viên là $a_{i}$ nên số cách chọn $\mathrm{C}$ là $a_{i}$. Với mỗi câu lạc bộ có $i$ thành viên số số cách chọn $x, y$ là $C_{i}^{2}$ nên $S=a_{i} C_{i}^{2}$ suy ra $a_{i} C_{i}^{2} \leq C_{n}^{2}$
Vậy $a_{i} \leq \frac{C_{n}^{2}}{C_{i}^{2}}$.
b) Ta có
$$
\begin{gathered}
m=\sum_{i=2}^{n} a_{i} \leq \sum_{i=2}^{n} \frac{C_{n}^{2}}{C_{i}^{2}}=\sum_{i=2}^{n} \frac{n(n-1)}{i(i-1)} \\
=n(n-1) \sum_{i=2}^{n}\left(\frac{1}{i-1}-\frac{1}{i}\right)=n(n-1)\left(1-\frac{1}{n}\right)=(n-1)^{2}
\end{gathered}
$$

Bài 19.

Cho 6 điểm nằm trong mặt phẳng và không có ba điểm nào thẳng hàng. Mỗi cạnh nối hai điểm được tô bởi hai màu xanh / đỏ. Hỏi có ít nhất bao nhiêu tam giác được tô cùng màu?

Lời giải

Gọi $S$ là số bộ $(A, B, C)$ sao cho cạnh $A B, A C$ được tô cùng màu.

Giả sử số tam giác được tô cùng màu là $n$ thì số tam giác được tô không cùng màu là $C_{6}^{3}-n=20-n$

Với mỗi tam giác cùng màu số bộ $(A, B, C)$ là $3 .$

Với mỗi tam giác không được tô cùng màu thì số bộ $(A, B, C)$ là 1 . Vậy $S=3 n+20-n=20+2 n$.

Với mỗi đỉnh $C$ là gọi $a_{1}$ là số cạnh màu đỏ nhận $C$ làm đầu mút và $a_{2}$ là số cạnh màu xanh nhận $C$ làm đầu mút. Ta có $a_{1}+a_{2}=5$.

Và $C_{a_{1}}^{2}+C_{a_{2}}^{2} \geq C_{3}^{2}+C_{2}^{2}=4$ nên $S \geq 6.4=24$.

Suy ra $20+2 n \geq 24$ nên $n \geq 2$.

Bài 20. (Ukraina TST 2013)

Cho đa giác lồi có 17 đỉnh $A_{1} A_{2} A_{3} \ldots A_{17}$ và với hai đỉnh $A_{i}, A_{j}$ bất kỳ trong số các đỉnh của đa giác, ta định hướng cho đoạn thẳng nối chúng để có vectơ: $\overrightarrow{A_{i} A_{j}}$ hoặc $\overrightarrow{A_{j} A_{i}}$. Sau khi thực hiện với mọi cặp đỉnh, gọi $S$ là số tam giác có tổng các vectơ đặt trên 3 cạnh là $\overrightarrow{0}$. Tim GTNN và GTLN của $S .$

Lời giải

Gọi số tam giác có tổng các vector trên 3 cạnh bằng $\overrightarrow{0}$ là $n$.
Ta gọi $S$ là số bộ 3 diểm $(A, B, C)$ trong 17 đỉnh mà có hai vector $\overrightarrow{A C}, \overrightarrow{B C}$ hoặc $\overrightarrow{\mathrm{CA}}, \overrightarrow{\mathrm{CB}}$ tức là với với các vector đặt trên hai cạnh $A C, B C$ thì $C$ cùng là điểm đầu hoặc cùng là điểm cuối.

Với mỗi tam giác mà tổng các vector đặt trên các cạnh bằng $\overrightarrow{0}$ thì số bộ như trên bằng 0 .

Với mỗi tam giác mà tổng các vector đặt trên các cạnh khác $\overrightarrow{0}$ thì số bộ như trên bằng 2 .

Vậy $S=2\left(C_{17}^{3}-n\right)=1360-2 n$

Với mỗi điểm $A_{i}$ ta gọi $d_{i}$ là số vector nhận $A_{i}$ là điểm đầu và $c_{i}$ là số vector nhận $A_{i}$ là điểm cuối.

Ta có $d_{i}+c_{i}=16$ và $C_{c_{i}}^{2}+C_{d_{i}}^{2} \geq \frac{16(16-2)}{2.2}=56$.

Nên $S \geq 56.17=952$

Suy ra $1360-2 n \geq 952 \Rightarrow n \leq 204$

Bài 21. (China MO 2007).

Trong một trường THPT, có 2007 học sinh nam và 2007 học sinh nữ. Mỗi học sinh tham gia không quá 100 CLB ở trường. Biết rằng hai học sinh bất kỳ khác giới tính đều có tham gia ít nhất 1 CLB chung nào đó. Chứng minh rằng có 1 CLB mà có ít nhất 11 thành viên nam và 11 thành viên nữ.

Lời giải

Ta chứng minh bằng phản chứng, giả sử mỗi câu lạc bộ đều có nhiều nhất 10 học sinh nam hoặc nhiều nhất 10 học sinh nữ. Gọi $S$ là số bộ $(A, B, C)$ mà học sinh nam $A$ và học sinh nữ $B$ tham gia câu lạc bộ $C$.

Vì mỗi học sinh nam và học sinh nữ bất kì tham gia ít nhất một câu lạc bộ nên $S \geq 2007^{2}$

Ta chia các câu lạc bộ thành 2 trường hợp.

TH1: Câu lạc bộ có không quá 10 nam. Số học sinh nữ là 2007 và mỗi học sinh tham gia không quá một câu lạc bộ nên số cặp $(B, C)$ trong trường hợp này không vượt quá: 2007.100 vì thế số bộ $(A, B, C)$ không vượt quá 10.2007.100.

TH2: Câu lạc bộ có không quá 10 nữ. Số bộ $(A, B, C)$ trong trường hợp này không quá 2007.10.100.

Suy ra: $S \leq 2.2007 .10 .100=2000.2007$

Vi vậy $2007^{2} \leq 2000.2007$. Vô lý.

Bài 22. (Trường Xuân 2013)

Cô giáo có tất cả $X$ viên kẹo gồm 11 loại kẹo khác nhau. Cô chia cho các học sinh của mình mỗi người một số viên kẹo và không có học sinh nào nhận nhiều hơn một viên kẹo ở cùng một loại kẹo. Cô yêu cầu hai học sinh khác nhau bất kì so sánh các viên kẹo mình nhận được và viết số loại kẹo mà cả hai cùng có lên bảng. Biết rằng mỗi cặp bất kì đều được lên bảng đúng một lần. Gọi tổng các số được viết lên bảng là $M$. Xác định giá trị nhỏ nhất của $M$.

a) Khi $X=2013$

b) Khi $X=2017$

Lời giải

Ta đánh số các loại kẹo từ 1 đến 11 giả sử số viên kẹo loại $i$ là $a_{i}$. Ta có $\sum_{i=1}^{11} a_{i}=X .$ Ta thấy $M=\sum_{i=1}^{11} C_{a_{i}}^{2}$

a) Khi $X=2013$ ta có $11 \mid 2013$

Ta thấy $M \geq \frac{2013(2013-11)}{2.11}=183183$ Đẳng thức xảy ra khi $a_{i}=\frac{2013}{11}=183$

b) Khi $X=2017$ ta có $2017=11.183+4$ nên

$M \geq 7 C_{183}^{2}+4 C_{184}^{2}=183915$

Bài 23. (IMO 1998)

Trong một cuộc thi có $a$ thí sinh và $b$ giám khảo, trong đó $b \geq 3$ là một số nguyên lẻ. Mỗi giám khảo đánh giá một thí sinh là “đạt” hoặc “trượt”. Giả sử $k$ là một số sao cho, với 2 giám khảo bất kì, đánh giá của họ là như nhau với nhiều nhất $k$ thí sinh. Chứng minh rằng

$$\dfrac{k}{a} \geq \frac{b-1}{2 b} $$

Lời giải

Gọi $T$ là số bộ $\left(\left\{g_{1}, g_{2}\right\}, s\right)$ trong đó giám khảo $g_{1}$ và $g_{2}$ có cùng đánh giá với thí sinh $s .$

Vi hai giám khảo bất kì có cùng đánh giá với nhiều nhất $k$ thí sinh nên $T \leq k C_{b}^{2}$

Với một thí sinh $s$ bất kì, giả sử có $p$ giám khảo đánh giá “đạt” và $q$ giám khảo đánh giá “trượt”. Ta có $p+q=b$ và số cặp giám khảo có cùng đánh giá với s là:

$$ C_{p}^{2}+C_{q}^{2}=\frac{p^{2}+q^{2}-p-q}{2} \geq \frac{\frac{(b-1)^{2}}{4}+\frac{(b+1)^{2}}{4}-b}{2}=\frac{(b-1)^{2}}{4} $$

Bài 24. (China TST 1992)

Có 16 học sinh tham gia một cuộc thi có $n$ câu hỏi và mỗi câu hỏi có 4 lựa chọn. Biết rằng 2 học sinh bất kì có không quá 1 câu trả lời chung cho tất cả các câu hỏi. Tìm giá trị lớn nhất của $n$.

Lời giải

Gọi $S$ là số bộ $\left(\left\{h_{1}, h_{2}\right\}, c\right)$ mà 2 học sinh $h_{1}$ và $h_{2}$ có cùng câu trả lời với câu hỏi $c$.

Vi 2 học sinh có không có quá một câu trả lời chung cho các các câu hỏi nên $S \leq C_{16}^{2}=120$

Với mỗi câu hỏi $c$ có bốn phương án trả lời, gọi số thí sinh chọn phương án thứ $i$ là $a_{i}$. Ta có $\sum_{1}^{4} a_{i}=16$

Khi đó số cặp thí sinh $\left\{h_{1}, h_{2}\right\}$ mà có cùng câu trả lời cho câu hỏi $c$ là:
$$ C_{a_{1}}^{2}+C_{a_{2}}^{2}+C_{a_{3}}^{2}+C_{a_{4}}^{2} \geq \frac{16(16-4)}{2.4}=24 $$
Ta có: $S \geq 24 n$.
Suy ra $24 n \leq 120$ hay $n \leq 5$.

Bài 25.

Một CLB có $n$ thành viên và họ đã tham gia vào 12 buổi chuyên đề, mỗi buổi có 24 thành viên tham gia. Biết rằng hai thành viên bất kỳ tham gia chung không quá một buổi.

a) Tìm giá trị nhỏ nhất của $n$;

b) Câu hỏi tương tự nếu có 10 buổi chuyên đề và mỗi buổi có 7 thành viên tham gia.

Lời giải

a) Trước hết ta chuyển đổi giả thiết của bài toán “hai thành viên bất kỳ tham gia chung không quá một buổi” thành “hai buổi bất kì có không quá một thành viên chung”.

Ta đánh số các thành viên từ 1 đến $n .$ Gọi $a_{i}$ số buổi chuyên đề mà thành viên thứ $i$ tham gia thì $\sum_{i=1}^{n} a_{i}=12.24=288$

Gọi $T$ là số bộ $(a, B, C)$ mà thành viên $a$ tham gia vào cả 2 buổi chuyên đề $B, C$. Vi hai buổi bất kì có không quá một thành viên chung nên $T \leq C_{12}^{2}=66$. Ta có $T=\sum_{i=1}^{n} C_{a_{i}}^{2} \geq \frac{288(288-n)}{2 n}$

Suy ra: $66 \geq \frac{288(288-n)}{2 n}$. Dẫn đến $n \geq 198$.

Ta thấy $\left\lfloor\frac{288}{198}\right\rfloor=1$ nên các số $a_{i}$ chỉ nhận giá trị bằng 1 hoặc 2 . Giả sử có $k$ số $a_{i}$ bằng 1 khi đó $T=k$ suy ra $k \leq 66$

Suy ra: $288=2 k+(n-k)=n+k \leq n+66$. Vậy $n=222$ là giá trị nhỏ nhất.

Khi đó ta có 66 số $a_{i}$ bằng 2 và 156 số $a_{i}$ bằng $1 .$

b) $\sum_{i=1}^{n} a_{i}=10.7=70$

Gọi $T$ là số bộ $(a, B, C)$ mà thành viên $a$ tham gia vào cả 2 buổi chuyên đề $B, C$. Vi hai buổi bất kì có không quá một thành viên chung nên $T \leq C_{10}^{2}=45$.

Ta có $T=\sum_{i=1}^{n} C_{a_{i}}^{2} \geq \frac{70(70-n)}{2 n}$

Suy ra: $45 \geq \frac{70(70-n)}{2 n}$. Dẫn dến $n \geq 31$

Ta thấy $\left\lfloor\frac{70}{31}\right\rfloor=2$ nên các số $a_{i}$ chỉ nhận giá trị bằng 2 hoặc $3 .$

Giả sử có $k$ số $a_{i}$ bằng 3 khi đó $T=n-k+3 k=n+2 k$ suy ra $n+2 k \leq 45$.

Suy ra: $70=3 k+2(n-k)=2 n+k=\frac{3 n}{2}+\frac{n}{2}+k \leq \frac{3 n}{2}+\frac{45}{2}$. Vậy $n \geq 32$.

Vậy giá trị nhỏ nhất của $n$ là 32 . Khi đó ta có $k=6$ số $a_{i}$ bằng 3 và 26 số $a_{i}$ bằng 2 .

Bài 26.

Trên mặt phẳng cho tập hợp $A$ gồm 66 điểm phân biệt và tập hợp $B$ gồm 16 đường thẳng phân biệt. Gọi $m$ là số bộ $(a, b)$ sao cho $a \in A, b \in$ $B$, $a \in b$. Chứng minh rằng $m \leq 159$.

Lời giải

Để $m$ đạt giá trị lớn nhất thì tất cả điểm của $A$ phải thuộc vào các đường thẳng của $B$. Ta đặt tên các điểm là $A_{1}, \ldots, A_{66} .$ Gọi số đường thẳng trong tập $B$ đi qua điểm $A_{i}$ là $a_{i}$. Ta có $\sum_{i=1}^{16}=m$.

Gọi $T$ là số bộ $\left(X, d_{i}, d_{j}\right)$ mà $X \in A$ là giao điểm của $d_{i}$ và $d_{j}$ và $d_{i}, d_{j} \in B$. Rõ ràng $T \leq C_{16}^{2}=8.15=120$
Mặt khác $T=\sum_{i=1}^{66} C_{a_{i}}^{2} \geq \frac{m(m-66)}{2.66}$.
Ta có: $120 \geq \frac{m(m-66)}{2.66}$. Suy ra $m \leq 163$.
Ta có $\left\lfloor\frac{m}{66}\right\rfloor \leq 2$ nên ta dự đoán giá trị nhỏ nhất của $m$ ra khi $a_{i}$ nhận các giá trị là 2 hoặc $3 .$
Giả sử có $k$ giá trị $a_{i}$ bằng 3 thì $m=3 k+2(66-k)=132+k$.
Và $T=3 k+(66-k)=66+2 k=2(k+132)-198=2 m-198$.
Suy ra: $2 m-198 \leq 120$ nên $m \leq 159$.
Đẳng thức xảy ra khi có 27 số $a_{i}$ nhận giá trị bằng 3 và 39 số $a_{i}$ nhận giá trị bằng 2 .

Nhận xét $2.5$. Bài toán này thực tế giống bài toán trên vì có một tính chất mà người giải bài toán phải tự nhận thấy là “hai đường thẳng phân biệt có không quá một điểm chung”.

Bài 26.

Trên mặt phẳng cho tập hợp $A$ gồm 66 điểm phân biệt và tập hợp $B$ gồm 16 đường thẳng phân biệt. Gọi $m$ là số bộ $(a, b)$ sao cho $a \in A, b \in$ $B$, $a \in b$. Chứng minh rằng $m \leq 159$.

Lời giải

Để $m$ đạt giá trị lớn nhất thì tất cả điểm của $A$ phải thuộc vào các đường thẳng của $B$. Ta đặt tên các điểm là $A_{1}, \ldots, A_{66} .$ Gọi số đường thẳng trong tập $B$ đi qua điểm $A_{i}$ là $a_{i}$. Ta có $\sum_{i=1}^{16}=m$.

Gọi $T$ là số bộ $\left(X, d_{i}, d_{j}\right)$ mà $X \in A$ là giao điểm của $d_{i}$ và $d_{j}$ và $d_{i}, d_{j} \in B$. Rõ ràng $T \leq C_{16}^{2}=8.15=120$
Mặt khác $T=\sum_{i=1}^{66} C_{a_{i}}^{2} \geq \frac{m(m-66)}{2.66}$.
Ta có: $120 \geq \frac{m(m-66)}{2.66}$. Suy ra $m \leq 163$.
Ta có $\left\lfloor\frac{m}{66}\right\rfloor \leq 2$ nên ta dự đoán giá trị nhỏ nhất của $m$ ra khi $a_{i}$ nhận các giá trị là 2 hoặc $3 .$
Giả sử có $k$ giá trị $a_{i}$ bằng 3 thì $m=3 k+2(66-k)=132+k$.
Và $T=3 k+(66-k)=66+2 k=2(k+132)-198=2 m-198$.
Suy ra: $2 m-198 \leq 120$ nên $m \leq 159$.
Đẳng thức xảy ra khi có 27 số $a_{i}$ nhận giá trị bằng 3 và 39 số $a_{i}$ nhận giá trị bằng 2 .

Bài 27. (IMO Shortlist 1995)

Một cuộc họp có $12 k$ người và mỗi người bắt tay với đúng $3 k+6$ người còn lại. Biết rằng với mọi cách chọn cặp hai người, số người bắt tay với cả hai là như nhau. Tính số người trong cuộc họp.

Lời giải

Gọi $S$ là số bộ $(a, b, c)$ mà $a$ bắt tay với cả $b, c$.

Nếu ta đếm theo $a$ thì $S=12 k C_{3 k+6}^{2}$. Với 2 người bất kì, thì số người bắt tay với cả 2 người không đổi, ta đặt bằng $m$.

Nếu ta đếm theo $(b, c)$ thì $S=m C_{12 k}^{2}$.

Suy ra $m=\frac{3(k+2)(3 k+5)}{12 k-1}=3 \frac{3 k^{2}+11 k+10}{12 k-1} \in \mathbb{Z}$.

Giải phương trình nghiệm nguyên ta được $k=3$ và $m=6$.

Vậy cuộc họp có 36 người.

Bài 28.

Cho số nguyên tố $p$. Trên bàn cờ hình vuông có cạnh $p^{2}+p+1$, tìm số lớn nhất các ô có thể tô màu sao cho không tồn tại hình chữ nhật có 4 đỉnh cùng màu và các cạnh song song với các cạnh của hình chữ nhật.

Lời giải

Giả sử ta tô màu $n$ ô.

Gọi $S$ là số bộ $\left(c_{i}, c_{j}, h_{k}\right)$ trong đó các cột $c_{i}$ và $c_{j}$ và giao với hàng $h_{k}$ tại hai ô được tô màu.

Để không xuất hiện hình chữ nhật nào có bốn đỉnh được tô màu thì với mỗi hai cột $c_{i}$ và $c_{j}$ thì có nhiều nhất một hàng $h_{k}$ nên $S \leq C_{p^{2}+p+1}^{2}$.

Với mỗi hàng, ta giả sử số ô được tô màu trên hàng đó là $a_{i}$ thì

$$ \sum_{i=1}^{p^{2}+p+a} a_{i}=n $$

Khi đó $S=\sum_{i=1}^{p^{2}+p+1} C_{a_{i}}^{2} \geq \frac{n\left(n-p^{2}-p-1\right)}{2\left(p^{2}+p+1\right)}$

Suy ra $C_{p^{2}+p+1}^{2} \geq \frac{n\left(n-p^{2}-p-1\right)}{2\left(p^{2}+p+1\right)}$

Từ đó ta thu được $n \leq(p+1)\left(p^{2}+p+1\right)$.

Đẳng thức xảy ra khi mỗi hàng có $p+1$ ô được tô màu và trên 2 cột bất kì chỉ có 2 ô nằm trên cùng một hàng được tô màu.

Việc chỉ ra một cách một cách tô màu thỏa mãn đẳng thức xin dành lại cho người đọc chuyên đề này.

3/ Ma trận liên thuộc

Bài 29. (Iran 1999)

Giả sử $C_{1}, C_{2}, \ldots, C_{n} \quad(n \geq 2)$ là các đường tròn bán kính 1 . Không có 2 đường tròn nào tiếp xúc. Bất kì một đường tròn nào cũng cắt ít nhất một đường tròn khác. Giả sử $S$ là tập hợp các điểm thuộc ít nhất 2 đường tròn (tập các giao điểm). Chứng minh rằng $|S| \geq n$.

Lời giải

Gọi tập các đường tròn là $\mathcal{T}$ Giả sử $|S|=s$

Với mỗi cặp $\left(d, C_{i}\right)$ mà $d \in S$ và đường tròn $C_{i}$ đi qua $d$.

Ta gọi $x\left(d, C_{i}\right)$ là số đường tròn đi qua điểm $d$.

Và $y\left(d, C_{i}\right)$ là số điểm mà đường tròn $C_{i}$ đi qua.

Với điểm $d$ giả sử có một đường tròn $C_{j}$ nữa đi qua điểm $d$ thì đường tròn $C_{j}$ và $C_{i}$ cắt nhau tại một điểm $d^{\prime}$ nữa khác $d .$

Suy ra $x\left(d, C_{i}\right) \leq y\left(d, C_{j}\right)$.

Vì vậy ta có: $s=\sum_{d \in S, C \in \mathcal{T} \atop d \in C} \frac{1}{x\left(d, C_{i}\right)} \geq \sum_{d \in S, C \in \mathcal{T}, C C} \frac{1}{y\left(d, C_{i}\right)}=n$

Bài 30.

Cho $S_{1}, S_{2}, \ldots, S_{m}$ là các tập con phân biệt của tập ${1,2, \ldots, n}$ sao cho $\left|S_{i} \cap S_{j}\right|=1$ với mọi $i \neq j$. Chứng minh rằng $m \leq n$.

Lời giải

Ta phản chứng giả sử $m>n$.

Ta thấy các tập $S_{i}$ đều phải khác rỗng và ta chỉ cần xét với $m \geq 2$. Lời giải này sẽ được chúng tôi trình bày thông qua Ma trận liên thuộc. Trước hết ta lập một bảng ô vuông gồm $m$ dòng, dòng thứ $i$ sẽ ứng với tập $S_{i}$ và $n$ cột, cột thứ $j$ sẽ ứng với phần tử $j$ trong tập $\{1,2, \ldots, n\}$. Ô ở vị trí giao của cột thứ $j$ và dòng thứ $i$ sẽ được điền vào giá trị $a_{i j}$. Trong đó

$$ a_{i j}=\left\{\begin{array}{l} 1 \text { nếu } j \in S_{i} \\ 0 \text { nếu } j \notin S_{i} \end{array}\right. $$

Bây giờ ta xét trường hợp $a_{i j}=0$. Nếu trên cột $j$ có một số 1 giả sử là $a_{k j}=1$ thì $\left|S_{k} \cap S_{i}\right|=1$ nên trên dòng $i$ cũng có một số 1 . Và ta thấy rằng quy tắc này là một đơn ánh. Do đó, ta gọi $R_{i}$ là tổng các số trên dòng thứ $i$ và $C_{j}$ là tổng các số trên cột thứ $j$ thì $C_{j}<R_{i}$.

Suy ra $m-C_{j}>n-R_{i}$ và $\frac{C_{j}}{m-C_{j}}<\frac{R_{i}}{n-R_{i}}$.

Gọi $M$ là tổng các số trên bảng. Thì

$$

\begin{gathered}

M=\sum_{j=1}^{n} C_{j}=\sum_{j=1}^{n}\left(m-C_{j}\right) \frac{C_{j}}{m-C_{j}}=\sum_{j=1}^{n}\left(\sum_{i=1}^{m}\left(1-a_{i j}\right)\right) \frac{C_{j}}{m-C_{j}}= \\

=\sum_{j=1}^{n} \sum_{i=1}^{m} \frac{\left(1-a_{i j}\right) C_{j}}{m-C_{j}}<\sum_{j=1}^{n} \sum_{i=1}^{m} \frac{\left(1-a_{i j}\right) R_{i}}{n-R_{i}}=\sum_{i=1}^{m} \frac{R_{i}}{n-R_{i}}\left(\sum_{j=1}^{n}\left(1-a_{i j}\right)\right) \\

=\sum_{i=1}^{m} \frac{R_{i}}{n-R_{i}}\left(n-R_{i}\right)=\sum_{i=1}^{m} R_{i}=M

\end{gathered}

$$

Vô lý nên $m \leq n$

4/ Lựa chọn giữa đếm bằng hai cách và nguyên lý Dirichlet

Bài 31. (Chọn ĐT Lương Thế Vinh, Đồng Nai)

Trong một thư viện người ta quan sát thấy được:

i) Mỗi ngày có 5 người đọc sách.

ii) Hai ngày bất kì thì số người đọc sách là $9 .$

Hãy tính xem trong 1 tháng có bao nhiêu người đến đọc sách. Biết tháng đó có 30 ngày.

Lời giải

Lời giải. Giả sử có $n$ người đến đọc sách. Gọi tập hợp những người tham gia đọc sách ngày thứ $i$ là $A_{i}$.

Giả sử $A_{1}=\left\{b_{1}, \ldots, b_{5}\right\}$. Mỗi phần tử của $A_{1}$ này phải thuộc vào các tập từ $A_{2}, \ldots, A_{30} .$ Gọi $b_{1}$ là người tham gia nhiều ngày đọc sách nhất thì theo nguyên lý Drichlet thì $b_{1} \geq 6$
Giả sử $b_{1}$ tham gia đọc sách vào cách ngày $A_{i_{1}}, \ldots, A_{i_{m}}$ với $m \geq 6$ Giả $A_{k}$ không chứa $b_{1} .$ Do $|B|=5$ nên tồn tại hai tập $A_{i}$ và $A_{j}$ mà $A_{i_{j}} \cap B=$ $A_{i_{h}} \cap B .$ Mặt khác $\left|A_{i_{j}} \cap A_{i_{h}}\right|=1$ nên $A_{i_{j}} \cap A_{i_{h}}=a_{1}$. Vô lý.
Suy ra $a_{1} \in A_{k} \quad \forall 1 \leq k \leq 30$
Vậy số người tham gia đọc sách là $n=30.4+1=121$.

Bài 32. (Chọn đội tuyển Việt Nam 2014)

Trong một kỳ thi có 100 thí sinh và 24 vị giám khảo. Mỗi thí sinh được hỏi bởi đúng 10 giám khảo. Chứng minh rằng có 7 giám khảo mà mỗi thí sinh đều được hỏi bởi ít nhất một trong số họ.

Lời giải

Ta chứng minh bằng phản chứng. Giả sử với 7 giám khảo bất kỳ luôn tồn tại một thí sinh không được hỏi bởi cả 7 giám khảo đó. Gọi $S$ là số bộ $\left(\left\{a_{1}, \ldots, a_{7}\right\}, h\right)$ mà $h$ không được hỏi bởi cả 7 giám thị $a_{1}, \ldots, a_{7}$.

Ta có $S \geq C_{24}^{7}$.

Với mỗi thí sinh thì có 14 giám khảo không hỏi thí sinh đó. Nên $S=100 C_{14}^{7}$. Suy ra $C_{24}^{7} \geq 100 C_{14}^{7}$. Điều này vô lý.

Tất nhiên bài tập này có thể giải bằng cách sử dụng nguyên lý Drichlet.

Bài tập tự luyện

Bài tập 33 (IMO 2001). Hai mươi mốt cô gái và 21 chàng trai đã tham gia vào một cuộc thi toán học. Cho biết rằng

i) mỗi thí sinh giải quyết nhiều nhất là sáu bài toán, và

ii) với mỗi cặp của một cô gái và một chàng trai, có ít nhất một bài đã được giải quyết bởi cả cô gái và chàng trai.

Chứng minh rằng có một bài được giải bởi ít nhất là ba cô gái và ít nhất là ba chàng trai.

Bài tập 34 (IMO 2005). Trong một cuộc thi toán học có 6 bài toán được đưa ra cho các thí sinh. Mỗi cặp bài tập được giải bởi nhiều hơn $\frac{2}{5}$ số thí sinh. Không có thí sinh nào giải được cả 6 bài. Chứng minh rằng có ít nhất 2 thí sinh mà mỗi người giải được đúng 5 bài.

Bài tập 35 (IMO 1987). Cho $p_{n}(k)$ là số hoán vị của tập ${1,2, \ldots, n}(n \geq 1)$ có đúng $k$ điể̉ cố định. Chứng minh rằng

$$ \sum_{k=1}^{n} k \cdot p_{n}(k)=n ! $$

Bài tập 36. Cho $S$ là một tập với $|S|=n$. Giả $S_{1}, \ldots, S_{m}$ là các tập con của $S$ sao cho $S_{i} \not \subset S_{j} \quad \forall i \neq j$. Chứng minh rằng $m \leq C_{n}^{\left[\frac{n}{2}\right]}$

Bài tập 37. Cho các tập hợp $S_{1}, \ldots, S_{m}$, mỗi tập có 3 phần tử. Và $S=S_{1} \cup$ $S_{2} \cup \ldots \cup S_{m}$. Thỏa mãn:

i) $\left|S_{i} \cup S_{j}\right|=1 \quad \forall i \neq j ;$

ii) $\forall x, y \in S, x \neq y$ thì tồn tại một tập $S_{i}$ mà $x, y \in S_{i}$.

Chứng minh rằng: $|S|=7$ và họ $S_{1}, \ldots, S_{m}$ tồn tại.

Bài tập 38 (China TST 1990). Cho 7 điểm trên mặt phẳng và ta vẽ các đường tròn qua đúng 4 điểm nào đó trong số chúng. Tính số đường tròn lớn nhất có thể có.

Các bài toán biến đổi góc cạnh – Bài tập

BÀI TẬP CÁC BÀI TOÁN BIẾN ĐỔI GÓC

 

Bài 1 Cho tam giác $ABC$ các đường cao cắt nhau tại $H$. Chứng minh rằng đường tròn Euler của các tam $ABH, ACH, BCH$ và $ABC$ là trùng nhau

Bài 2 Cho tứ giác $ABCD$. Chứng minh rằng đường tròn Euler của các tam giác $ABC, ACD, ABD, BCD$ cùng đi qua một điểm.

Bài 3 Cho tứ giác $ABCD$ nội tiếp. Gọi $d_a$ là đường thẳng simson của tam giác $BCD$ ứng với điểm $A$; các đường thẳng $d_b, d_c, d_d$ được định nghĩa tương tự. Chứng minh rằng các đường thẳng $d_a, d_b, d_c, d_d$ đồng quy.

Bài 4 Cho hai điểm $P, Q$ thuộc miền trong của tam giác $ABC$ sao cho $$\angle ACP = \angle BCQ, \angle CAP = \angle BAQ$$ Gọi $D, E, F$ là hình chiếu vuông góc của $P$ trên các đường thẳng $BC, AC, AB$. Chứng minh rằng nếu $\angle DEF = 90^\circ$ thì $Q$ là trực tâm của tam giác $BDF$.

Bài 5(IMO 2007) Xét 5 điểm $A, B, C, D, E$ sao cho $ABCD$ là hình bình hành và $B, C, D, E$ cùng thuộc một đường tròn. Gọi $d$ là đường thẳng qua $A$, giả sử $d$ cắt đoạn $BC$ tại $F$ và $BC$ tại $G$. Giả sử $EF = EG = EC$, chứng minh rằng $d$ là phân giác góc $\angle DAB$.

Bài 6(VMO 2009) Trong mặt phẳng cho hai điểm $A$ và $B$ cố định ($A$ khác $B$). Một điểm $C$ di động trên mặt phẳng sao cho $\angle ACB = \alpha (0^o < \alpha < 180^o)$. Đường tròn tâm $I$ nội tiếp tam giác $ABC$ và tiếp xúc với $AB, BC, CA$ lần lượt tại $D, E, F$. $AI, BI$ cắt $EF$ tại $M, N$.

a) Chứng minh $MN$ có độ dài không đổi.
b) Chứng minh rằng đường tròn ngoại tiếp tam giác $DMN$ luôn đi qua một điểm cố định khi $C$ lưu động.

Bài 7 Cho tam giác nhọn $ABC$ nội tiếp đường tròn $(O)$. Các đường cao $AD$ và $BD$. Gọi $M$ là trung điểm $AB$, phân giá trong góc $\angle BCA$ cắt $DE$ tại $P$ và cắt $(O)$ tại $Q$. Gọi $C’$ là điểm đối xứng của $C$ qua $AB$. Tính $\angle C$ biết rằng 4 điểm $M, P, Q$ và $C’$ cùng thuộc một đường tròn.

Bài 8 Cho tam giác $ABC$, $M$ là trung điểm $BC$. Trên đoạn $AM$ lấy điểm $P$. Gọi $D$ là hình chiếu của $P$ trên $BC$. $E$ là một điểm thuộc đoạn $PD$. Gọi $H, K$ là hình chiếu của $E$ trên $AB, AC$. Chứng minh rằng $H, P, K$ thẳng hàng khi và chỉ khi $\angle EAB = \angle EAC$.

Bài 9 Cho tam giác $ABC$ với $I$ là tâm đường tròn nội tiếp. Gọi $K, L$ lần lượt là trực tâm các tam giác $IBC$ và $IAC$. Gọi $T$ là tiếp điểm của đường tròn bàng tiếp góc $C$ với cạnh $AB$. Chứng minh rằng $CT$ và $KL$ cắt nhau tại một điểm thuộc đường tròn $(I)$.

Bài 10 Cho đoạn thẳng $AB$ và điểm $C$ thuộc đoạn $AB (AC < BC)$. Đường tròn $w$ tâm $O$ thay đổi tiếp xúc với $AB$ tại $C$. Từ $A$ và $B$ vẽ các tiếp tuyến $AD$ và $BE$ ($D, E$là hai tiếp điểm khác $C$). $AD$ và $BE$ cắt nhau tại $P$.

a) Chứng minh rằng $DE$ luôn đi qua một điểm cố định
b) Gọi $F$ là giao điểm của $OC$ và $DE$. Chứng minh $PF$ luôn đi qua một điểm cố định.

Bài 11 Cho đường tròn $(O)$ và điểm $P$ nằm ngoài đường tròn. Vẽ các tiếp tuyến $PA, PB$ đến $(O)$ với $A, B$ là các tiếp điểm. $C$ là điểm trên cung nhỏ $AB$, tiếp tuyến tại $C$ cắt $PA, PB$ và $PO$ lần lượt tại $D, E, F$. Chứng minh rằng đường tròn ngoại tiếp các tam giác $PAB, PDE$ và $PCF$ cùng đi qua một điểm khác $P$.

Bài 12(Chọn đội tuyển Toán Việt Nam năm 2000) Cho hai đường tròn $(C_1)$ và $(C_2)$ cắt nhau tại $P$ và $Q$. Tiếp tuyến chung (tiếp tuyến gần $P$) tiếp xúc với $(C_1)$ tại $A$ và tiếp xúc với $(C_2)$ tại $B$. Tiếp tuyến của $(C_1)$ và $(C_2)$ tại $P$ cắt hai đường tròn tại $E$ và $F$ (khác $P$). Gọi $H$ và $K$ là các điểm trên tia $AF$ và $BE$ sao cho $AH = AP$ và $BK = BP$. Chứng minh rằng $A, H, Q, K, B$ cùng thuộc một đường tròn.

Bài 13(IMO 2009) Cho tam giác $ABC$ cân tại $A$. Phân giác trong góc $A$ và $B$ cắt $BC$ và $AC$ lần lượt tại $D$ và $E$. Gọi $K$ là tâm đường tròn nội tiếp tam giác $ACD$. Cho $\angle BEK = 45^o$. Tìm tất cả các giá trị của $\angle BAC$.

Bài 14 Cho tam giác $ABC$ ngoại tiếp đường tròn tâm $I$. Trên các đoạn $AI, BI$ và $CI$ lấy các điểm $A’,B’,C’$. Đường trung trực của các đoạn $AA’, BB’, CC’$ đôi một cắt nhau tại $A_1, B_1, C_1$. Chứng minh rằng tâm đường tròn ngoại tiếp của tam giác $ABC$ và tam giác $A_1B_1C_1$ trùng nhau khi và chỉ khi $I$ là trực tâm của tam giác $A’B’C’$.

Bài 15 (IMO 2017) Cho $R,S$ là hai điểm phân biệt trên đường tròn $\Omega$ sao cho $RS$ không phải đường kính. Gọi $d$ là tiếp tuyến của $\Omega$ tại $R$. Lấy điểm $T$ sao cho $S$ là trung điểm của đoạn thẳng $RT$. Lấy điểm $J$ trên cung nhỏ $RS$ của $\Omega$ sao cho $(JST)$ cắt $d$ tại hai điểm phân biệt. Gọi $A$ là giao điểm gần $R$ nhất của $d$ và $(JST)$. $AJ$ cắt lại $\Omega$ tại $K$. Chứng minh $KT$ tiếp xúc với $(JST)$.

Bài 16(Đề thi HSG Bulgari năm 2016) Cho tam giác $ABC$ cân tại $C$, trên tia đối của tia $CA$ lấy điểm $D$ sao cho $AC > CD$. Phân giác $\angle BCD$ cắt $BD$ tại $N$. $M$ là trung điểm $BD$, tiếp tuyến tại $M$ của $(AMD)$ cắt $BC$ tại $P$. Chứng minh rằng 4 điểm $A, P, M, N$ cùng thuộc một đường tròn.

Bài 17(Đề thi HSG Iran 2018 – Vòng 3) Cho tam giác $ABC$, đường tròn $w$ thay đổi qua $B, C$ cắt các cạnh $AB, AC$ tại $E$ và $F$. $BF, CE$ cắt $(ABC)$ tại $B’, C’$. $A’$ là điểm thuộc $BC$ sao cho $\angle C’A’B = \angle B’A’C$. Chứng minh rằng đường tròn ngoại tiếp tam giác $A’B’C’$ luôn đi qua một điểm cố định.

Bài 18(IMO shortlist 2017) Cho tam giác $ABC$ nhọn nội tiếp đường tròn tâm $O$. Đường thẳng $OA$ cắt đường cao từ $B$ và $C$ của tam giác $ABC$ lần lượt tại $P$ và $Q$. $H$ là trực tâm tam giác $ABC$. Chứng minh rằng tâm đường tròn ngoại tiếp tam giác $HPQ$ thuộc đường trung trung tuyến của tam giác $ABC$.

Bài 19 Cho tam giác $ABC$ nội tiếp đường tròn $(O)$. Tiếp tuyến tại $A$ và $B$ cắt nhau tại $N$, tiếp tuyến tại $B$ và $C$ của $(O)$ cắt nhau tại $P$; tiếp tuyến tại $A$ và $C$ cắt nhau tại $M$.
a) Chứng minh $PA, CN$ và $BM$ đồng quy tại một điểm $L$.
b) Gọi $X, Y, Z$ là hình chiếu của $L$ trên $BC, AC$ và $AB$. Chứng minh $L$ thuộc đường thẳng Euler của tam giác $XYZ$.
c) Gọi $A’, B’, C’$ là trung điểm của $OP, OM$ và $ON$. Chứng minh rằng các đường thẳng $AA’, BB’$ và $CC’$ đồng quy.

Bài 20 Cho tam giác $ABC$ có các đường cao $AD, BE, CF$ cắt nhau tại $H$. Đường tròn đường kính $BH$ cắt $DE$ tại $K$, đường tròn đường kính $CH$ cắt $DF$ tại $L$. Chứng minh $KL$ vuông góc với đường thẳng euler của tam giác $ABC$.

Bài 21 Cho tam giác $ABC$ có $\angle A = 45^o$. Các đường cao $AD, BE, CF$. Gọi $A’, B’, C’$ lần lượt là hình chiếu của $A, B, C$ trên $EF, DF, DE$. Chứng minh rằng tâm đường tròn ngoại tiếp của tam giác $A’B’C’$ thuộc đường tròn euler của tam giác $ABC$.

Bài 22 Cho tam giác $ABC$, đường thẳng $d$ cắt các cạnh $AB, AC$ tại $D, E$ và đường thẳng $BC$ tại $F$. Gọi $O,O_a, O_b, O_c$ lần lượt là tâm đường tròn ngoại tiếp các tam giác $ABC, ADE, BDF, CEF$.

a) Chứng minh rằng 4 điểm $O, O_a, O_b, O_c$ cùng thuộc một đường tròn.
b) Chứng minh trực tâm tam giác $O_aO_bO_c$ thuộc $d$.

Bài 23(IMO 2019) Cho tam giác $ABC$, các điểm $A_1$ thuộc cạnh $BC$ và $B_1$ thuộc cạnh $AC$. Trên đoạn $AA_1, BB_1$ lấy $P, Q$ sao cho $PQ$ song song $AB$. Trên tia $PB_1$ lấy $P_1$ sao cho $\angle PP_1C = \angle BAC$, trên tia $QA_1$ lấy điểm $Q_1$ sao cho $QQ_1C = \angle ABC$. Chứng minh 4 điểm $P, Q, P_1, Q_1$ đồng viên.

Bài 24 Cho tứ giác $ABCD$ nội tiếp đường tròn tâm $O$. Các đường phân giác trong của các góc $A, B, C, D$ cắt nhau tạo thành tứ giác nội tiếp tâm $I$. Các đường phân giác ngoài cắt nhau tạo thành tứ giác nội tiếp tâm $J$. Chứng minh rằng $O$ là trung điểm của $IJ$.

Bài 25 Cho tứ giác $ABCD$ nội tiếp đường tròn tâm $O$, $AD$ và $BC$ cắt nhau tại $K$. Đường tròn ngoại tiếp tam giác $KAC$ và $KBD$ có tâm là $I$ và $J$ cắt nhau tại $M$. Chứng minh
a) $O, J, I, M$ đồng viên.
b) $OM \bot KM$.

Bài 26 Cho tam giác $ABC$ nội tiếp đường tròn $w$. Trung tuyến $BM$ và $CN$ cắt $w$ tại $D$ và $E$. Đường tròn tâm $O_1$ qua $D$ và tiếp xúc với $AC$ tại $C$; đường tròn $O_2$ qua $E$ và tiếp xúc với $AB$ tại $B$.

a) Chứng minh rằng $O_1 O_2$ qua tâm đường tròn euler của tam giác $ABC$.
b) Gọi $K$ là giao điểm của $O_1M$ và $O_2N$. Chứng minh rằng $AK\bot BC$.

 

Bài 27 (IMO Shorlist 2019) Cho tam giác $ABC$, đường tròn $w$ qua $A$ cắt các cạnh $AB, AC$ tại $D$ và $E$ tương ứng; $w$ cắt $BC$ tại $F$ và $G$ sao cho $F$ nằm giữa $B$ và $G$. Tiếp tuyến tại $F$ của $(BDF)$ và tiếp tuyến tại $G$ của $(CEG)$ cắt nhau tại $T$. Giả sử $A, T$ phân biệt. Chứng minh rằng $AT$ song song $BC$.

Bài 28 (ISL 2107) Cho tam giác $ABC$ khác tam giác cân. Các đường cao từ $B$ và $C$ cắt nhau tại $H$. Đường thẳng $OA$ cắt $BH, CH$ tại $P$ và $Q$. Chứng minh rằng tâm đường tròn ngoại tiếp tam giác $HPQ$ thuộc trung tuyến của tam giác $ABC$.

 

Bài 29 (ISL 2015 – G2) Cho tam giác $ABC$ nội tiếp đường tròn $(O)$. Đường tròn $w$ tâm $A$ cắt cạnh $BC$ tại $D, E$ sao cho $D$ nằm giữa $B$ và $E$; $w$ cắt $(O)$ tại $F$ và $G$, trong đó $F$ thuộc cung nhỏ $AB$. Đường tròn ngoại tiếp tam giác $BDF$ cắt $AB$ tại $K$; đường tròn ngoại tiếp tam giác $CEG$ cắt $AC$ tại $L$. Gọi $X$ là giao điểm của $FK$ và $GL$. Chứng minh $A, X, O$ thẳng hàng.

Bài 30 (IMO 2013 – G6) Cho tam giác $ABC$, gọi $A_1$ là tiếp điểm của đường tròn bàng tiếp góc $A$ với $BC$; các điểm $B_1, C_1$ được xác định tương tự. Giả sử tâm đường tròn ngoại tiếp tam giác $A_1B_1C_1$ thuộc đường tròn ngoại tiếp tam giác $ABC$. Chứng minh tam giác $ABC$ vuông.